Vous êtes sur la page 1sur 56

USMLE WORLD STEP 2 CK (INTERNAL MEDICINE) *Block

1*

BLOCK # 1
1 Internal Medicine Cardiology 24 Internal Medicine Respiratory
2 Internal Medicine ENT 25 Internal Medicine Dermatology
3 Internal Medicine Cardiology 26 Internal Medicine Hem&Onc
4 Internal Medicine Cardiology 27 Internal Medicine Neurology
5 Internal Medicine Biostatistics 28 Internal Medicine Cardiology
6 Internal Medicine Neurology 29 Internal Medicine Rheumatology
7 Internal Medicine Hepatology 30 Internal Medicine Respiratory
8 Internal Medicine Hem&Onc 31 Internal Medicine ID
9 Internal Medicine Hepatology 32 Internal Medicine Cardiology
10 Internal Medicine Cardiology 33 Internal Medicine ID
11 Internal Medicine Miscellaneous 34 Internal Medicine Ophthalmology
12 Internal Medicine ID 35 Internal Medicine Endocrinology
13 Internal Medicine Genitourinary 36 Internal Medicine Hepatology
14 Internal Medicine Rheumatology 37 Internal Medicine Cardiology
15 Internal Medicine Hem&Onc 38 Internal Medicine Neurology
16 Internal Medicine Cardiology 39 Internal Medicine Hem&Onc
17 Internal Medicine Hepatology 40 Internal Medicine Cardiology
18 Internal Medicine Cardiology 41 Internal Medicine Cardiology
19 Internal Medicine Hepatology 42 Internal Medicine Genitourinary
20 Internal Medicine ENT 43 Internal Medicine Genitourinary
21 Internal Medicine Rheumatology 44 Internal Medicine Cardiology
22 Internal Medicine Hem & Onc 45 Internal Medicine ID
23 Internal Medicine Biostatistics 46 Internal Medicine ID

__________________________________________________________ 1
USMLE WORLD STEP 2 CK (INTERNAL MEDICINE) *Block
1*
Q NO 1: A 56-year-old female comes to your office for the evaluation of
fatigue and exercise intolerance. She also complains of occasional
palpitations. She denies any chest pain, syncope, fever or cough. She
has no bowel or urinary complaints. There is no recent change in her
appetite or weight. She denies any emotional stressors. She is taking
estrogen replacement therapy for her postmenopausal symptoms. She
is a non-smoker and non-alcoholic. Her vitals are stable and she is
afebrile. Her lungs are clear to auscultation. Cardiac auscultation reveals
a soft S1 and a blowing high-pitched pansystolic murmur at the apex
that radiates to the axilla. Chest x-ray shows mild enlargement of her
cardiac silhouette and clear lung fields. EKG is normal. Which of the
following is the most likely cause of her valvular dysfunction?

A. Rheumatic heart disease


B. Dilated cardiomyopathy
C. Infective endocarditis
D. Hypertrophic cardiomyopathy
E. Mitral annulus calcifications due to aging
F. Ischemic heart disease
G. Mitral valve prolapse

Explanation:
There are numerous causes of mitral regurgitation. Mitral valve
prolapse is the most common cause of isolated mitral regurgitation in
North America. All of the remaining options may cause mitral
regurgitation but they do so much less frequently in this country.
Rheumatic heart disease is the cause for mitral regurgitation in one
third of cases and it occurs more commonly in males.
Marked dilatation of left ventricle, due to any cause, may lead to MR.
The proposed mechanism is the dilatation of mitral annulus and
displacement of the papillary muscles.
Infective endocarditis may cause acute MR by causing damage to the
valve leaflets or to the chordatendinea. Hypertrophic cardiomyopathy
may cause mitral regurgitation by systolic anterior motion of the
mitral valve leaflet.
Mitral annuls calcification, due to aging, usually occurs in elderly
women. It is a degenerative process and it may result in severe MR.
Ischemia of the papilla muscle may cause transient MR. Ml may cause
acute mitral regurgitation by causing a rupture of the papillary
muscle.

Educational Objective:
Mitral valve prolapse is the most common cause of isolated mitral
regurgitation in North America.

2 __________________________________________________________
USMLE WORLD STEP 2 CK (INTERNAL MEDICINE) *Block
1*
Q NO 2: A 45-year-old Asian male presents with complaints of progressively
worsening sore throat and difficulty swallowing for the past 24 hours. You
notice that his voice is muffled and he has saliva drooling from his mouth.
He also has harsh shrill creaking noise associated with respiration. On
examination, few cervical lymph nodes are palpable with tenderness on
palpation over larynx. His vital signs are: PR is 106/min; BP is 120/80mm of
Hg; RR is 22/min; and Temperature is 39.3 C (103F). Which of the following
organisms are the most common cause of this condition?

A. H. Influenzae and Group A Streptococcus.


B. Mycobacterium and Herpes simplexvirus.
C. H. influenzae and Candida.
D. Group A streptococcus and Klebsiella.
E. Group A Streptococcus and Staphylococcus aureus.

Explanation:
Epiglottitis is inflammation and cellulitis of the soft tissue above
the vocal cords glottis which may cause life threatening airway
obstruction. Epiglottitis presents with rapid onset and progression of
symptoms as in this patient. It presents with sore throat, dysphagia,
drooling, muffled voice, and cough. The patient assumes a tripod
position, sitting up on hands with the tongue protruding out and head
leaning forward. Examination reveals cervical lymphadenopathy. Stridor
(harsh shrill noise with respiration.), laryngeal tenderness, and
respiratory distress are the commonly associated signs and symptoms.
Some common causes of epiglottitis are as follows:
H. Influenza type B is the most common cause in children and adults.
Streptococci group A is the second most common cause in adults.
Streptococcus pneumoniae.
H. Parainfluenza.
K. Pneumoniae.
Candida albicans.
Staphylococcus aureus.
N. Meningitidis.
Varicella zoster.

H. influenza and group A streptococcus are the most common causes of


epiglottitis in adults. Incidence is particularly higher in countries
where they do not vaccinate against H. Influenza. Indirect laryngoscopy
is considered to be the best for diagnosis. Immediate intubation may be
required in patients with severe airway obstruction. Antibiotics, anti
pyretics, racemic epinephrine, and steroids are the drugs of choice.
Regardless of the cause, all incidences of epiglottitis are considered
serious medical emergencies.
Differential diagnosis of epiglottitis consists of croup, angioedema,
candidiasis, anaphylaxis, diphtheria, foreign body aspiration,
peritonsillar abscess, and pharyngitis.
(Choice B, C, D and E) Mycobacterium, herpes simplexvirus, candida,
Klebsiella and staphylococcus aureus are rare causes of epiglottitis.

Educational Objective:
H. influenzae and Group A Streptococcus are the most common causes of
epiglottitis.

__________________________________________________________ 3
USMLE WORLD STEP 2 CK (INTERNAL MEDICINE) *Block
1*
Q NO 3: A 36-year-old female presents to the emergency room complaining
of chest pain that started suddenly while she was shopping at the mall. She
also reports shortness of breath, palpitations and diaphoresis. The pain is
retrosternal and radiates to the left arm. There are no aggravating or
relieving factors. On review of systems, the patient reports having had a
runny nose, sore throat and dry cough for the past 3 days. Her past medical
history is significant for panic attacks for which she takes paroxetine, and
dysfunctional uterine bleeding, for which she takes estrogen. Her family
history is significant for the sudden death of her father at age 44 from a
heart attack. Social history reveals that she has smoked one pack of
cigarettes per day for the past 15 years. Which of the following is the most
appropriate initial therapy for this patient?

A. Lorazepam
B. Heparin
C. Aspirin
D. Ibuprofen
E. Acetaminophen
F. Oxycodone

Explanation:
This vignette describes a 36-year-old woman with a hi stow of panic
attacks who presents with sudden-onset chest pain, dyspnea, and
diaphoresis. Despite her psychiatric history and relatively young age,
a thorough evaluation for an acute coronary event is necessary because
she has multiple cardiac risk factors. First, she has a positive family
history for acute coronary syndrome at a young age. Additionally, she
has a longstanding personal history of smoking and is taking estrogen
therapy, factors which together dramatically increase her risk of
thrombosis. This risk is increased further given that she is > 35 years
of age.
Myocardial infarction is probable in this patient; thus aspirin should
be administered as soon as possible. Aspirin therapy has been shown to
greatly decrease the mortality associated with acute coronary events.
Even if this episode turns out not to have been an acute coronary
event, one aspirin would not be detrimental.
(Choice B) Pulmonary embolism is possible, especially in light of this
patients risk factors for thrombosis (smoking, estrogen treatment,
age). However, her clinical presentation is somewhat atypical thus
further work-up is required before heparin can be administered.

Educational Objective:
Chest pain in a young person with cardiovascular risk factors warrants
a thorough cardiac work-up. Aspirin should be the first drug
administered when suspicion of a coronary artery event is high due to
its ability to prevent platelet aggregation and coronary spasm.

4 __________________________________________________________
USMLE WORLD STEP 2 CK (INTERNAL MEDICINE) *Block
1*
Q NO 4: A 53-year-old Caucasian woman comes to the physician for a
routine health maintenance examination. Recently, she noticed a mild
hand tremor while performing some fine tasks, like pouring out the
tea. She says that the tremor disappears for a while after drinking a
glass of wine. Her past medical history is insignificant. Her blood
pressure is 160/90 mmHg and heart rate is 80/min. Physical
examination is insignificant. Which of the following medications is the
best choice to treat this patients hypertension?
A. Propranolol
B. Verapamil
C. Amlodipine
D. Hydrochlorothiazide
E. Enalapril

Explanation:
This patient presents with hypertension and a benign essential tremor.
An essential tremor is characterized by a postural tremor (not a
resting tremor unlike Parkinsons disease) and usually disturbs the
performance of fine motor tasks. Sometimes, it runs in families with
autosomal-dominant inheritance. The pathophysiologic basis of this
condition is unclear. The inhibition of the tremor by a small amount of
alcohol is typical. Propranolol, a non-selective beta-blocker, is
effective in reducing the tremor probably by blocking beta-2 receptors.
Therefore, administering propranolol to this patient would help to
control both the hypertension and the essential tremor.
Other anti-hypertensive medications, like thiazide diuretics (Choice
D), calcium antagonists (Choices B and C), and ACE inhibitors (Choice
E), do not affect a benign essential tremor.

Educational Objective:
Propranolol is the drug of choice in patients who present with
hypertension and a benign essential tremor.

__________________________________________________________ 5
USMLE WORLD STEP 2 CK (INTERNAL MEDICINE) *Block
1*
Q NO 5: Consecutive readings of pulmonary capillary wedge pressure
(PCWP) were obtained from a patient in the Intensive Care Unit (ICU)
using a Swan-Ganz catheter. The readings are 20 mmHg, 22 mmHg,
21 mmHg, 22 mmHg, and 18 mmHg. Which of the following is the
median of the values given above?
A. 18 mmHg
B. 20 mmHg
C. 21 mmHg
D. 22 mmHg
E. 20.6 mmHg

Explanation:
It is important to know the difference between the measures of central
tendency. The median of a dataset is the number that divides the right
half of the data from the left half. In this case, 21 mmHg is in the
middle of the dataset; therefore, it is the median. If the number of
observations is even, finding the median becomes tricky. You should
find the middle two values, add them together and divide by two.
(Choices A and B) are not measures of the center in this dataset.
(Choice E) To find the mean of a dataset, you should add all the
observations and divide that sum by the number of observations. In this
case, the mean is equal to 20.6 mmHg.
(Choice D) Another measure of the center of a dataset is the mode.
Finding the mode is the easiest. The mode is the most frequent value of
a dataset. In the scenario described, the mode is 22 mmHg.

Educational Objective:
The median is the value that is located in the middle of a dataset. It
divides the right half of the data from the left half.

6 __________________________________________________________
USMLE WORLD STEP 2 CK (INTERNAL MEDICINE) *Block
1*
Q NO 6: A 12-year-old male child comes to the office after being referred
for a medical evaluation. His school teacher says that he has a problem
concentrating during class. He stares in space for a few seconds several
times a day, and appears totally absorbed in his thoughts. He is not
disruptive in class, but appears forgetful. There is no history of trauma,
infection or problems at birth. On examination, the child is alert with
stable vital signs. There is no loss of motor or sensory perception. Which
of the following can confirm the patients diagnosis?

A. CT scan of the head


B. EMG studies
C. EEG studies
D. Psychiatric evaluation
E. Lumbar puncture

Explanation:
Absence (petit mal) seizures are characterized by a sudden cessation,
without warning, of ongoing mental activity. Each episode rarely lasts
longer than 30 seconds. Minor motor symptoms are common, but complex
automations and clonic activities do not occur. The return of
consciousness is abrupt, and there is no postictal somnolence or
confusion. The condition is diagnosed best with EEG studies. An EEG
with activation procedures (hyperventilation, photic stimulation,
sleep) helps in further diagnosis and classification of seizures.
(Choice A) CT scan is generally not done for all seizures. CT scan is
recommended if there is suspicion of an associated tumor, bleed or
aneurysm, or if the patient has an unexplained first seizure. For those
with unexplained first seizures and normal CT findings, a follow-up
study at 3-6 months is advisable. In this case, the characteristic
history of petit mal seizures does not warrant a CT scan.
(Choice B) EMG studies are used to diagnose peripheral nerve disorders.
EMG is used to analyze the neuromuscular system, differentiate diseases
of neuromuscular system from primary neuropathies. It does not help in
the evaluation of seizure disorders.
(Choice D) At this point, a psychiatric evaluation is not the best next
step in management. There have been many instances in which petit mal
was misdiagnosed for a psychiatric disorder, and patients were treated
erroneously with medications. For these reasons, it is essential that
this child be thoroughly evaluated for a neurologic disorder first.
(Choice E) A lumbar puncture is indicated in all patients when an
infection is being suspected. In the above patient with a classic
presentation of petit mal seizures, a lumbar puncture is not indicated

Educational Objective:
Petit mal seizures are characterized by a sudden cessation of mental
activity. An episode is very short, but may occur repeatedly throughout
the day. There are no associated complex automatisms or tonic-clonic
activity. The diagnosis is best confirmed by EEG studies.

__________________________________________________________ 7
USMLE WORLD STEP 2 CK (INTERNAL MEDICINE) *Block
1*
Q NO 7: A 45-year-old Mexican male comes with fever, anorexia, and right
upper quadrant pain. For the past 12 days. The pain radiates to the right
shoulder tip and is aggravated by deep inspiration. The patient gives the
history of bloody diarrhea six months ago, after he returned from a trip to
Mexico. He has no other major illnesses and never had any surgery. He
denies any close contacts with animals, is a non-alcoholic, non-smoker, and
with a single monogamous sexual partner. On examination, he has right
upper quadrant abdominal and intercostal tenderness, but no jaundice. An
ultra sonogram of abdomen shows a single thin wall uniform cyst on the
superior surface of the right lobe of his liver. What is the most likely
diagnosis in this patient?

A. Pyogenic liver abscess


B. Amebic liver abscess
C. Hydatid cyst
D. Liver carcinoma
E. Hepatic adenoma

Explanation:
This patient has an amebic liver abscess. The history of travel to an
endemic area followed by dysentery and right upper quadrant pain with a
single cyst in right lobe of liver is indicative of amebic liver
abscess. Amebiasis is a protozoal disease caused by Entamoeba
histolytica. The primary infection is in the colon leading to bloody
diarrhea; however the ameba may be transported to the liver by portal
circulation thus leading to an amebic liver abscess (Choice B).
Amebic liver abscesses are generally single and are located usually in
the right lobe. An abscess on the superior surface of liver can cause a
pleuritic-like type of pain and the radiation to the shoulder. The
diagnosis can be made by stool examination for trophozoites, serology,
and liver imaging. The aspirate from the amebic liver abscess is
usually sterile. Treatment is with metronidazole.
(Choice A) Pyogenic liver abscess should be considered in the
differential diagnosis of amebic liver abscess. But this patients
history is more suggestive of amebic liver abscess. Pyogenic liver
abscess is generally secondary to surgery, a gastrointestinal
infection, or acute appendicitis. The condition of the patient would be
more severe than in amebic liver abscess.
(Choice C) A hydatid cyst is usually caused by an infection with
Echinococcus granulosus, which is acquired by intimate contacts with
dogs.
(Choice D) Unilocular cystic lesion as a presentation of hepatic
adenocarcinoma is not common. Cystic carcinoma generally has thick
irregular and hyper vascular walls with numerous septa. The other
inflammatory features are unlikely to be present with carcinoma.
(Choice E) Hepatic adenoma is a solid lesion and not a cystic lesion.

Educational Objective:
Entamoeba histolytica is a protozoan, which can cause amebic liver
abscess. Remember the Mexico trip.

8 __________________________________________________________
USMLE WORLD STEP 2 CK (INTERNAL MEDICINE) *Block
1*
Q NO 8: A 25-year-old African American woman presents with a photo
distributed skin rash and arthralgias. She is found to have low-range
proteinuria and abnormal urinary sediment. Renal biopsy findings are
consistent with focal proliferative glomerulonephritis. Her complete
blood count shows:
Erythrocyte count 3.2 mln/mm
Platelets 60,000/mm
Leukocyte count 2500/mm
Which of the following is the most likely cause of these hematologic
findings?

A. Bone marrow hypoplasia


B. Ineffective hemopoiesis
C. Abnormal pooling of blood cells
D. Peripheral destruction of blood cells
E. Dilutional pancytopenia

Explanation:
The patient described in this clinical vignette displays a number of
symptoms characteristic for systemic lupus erythematosus (SLE). This
chronic autoimmune disorder affects mostly women of childbearing age.
Black women have a higher risk of developing SLE than Caucasian and
Asian women.
Hematologic abnormalities are common in patients with SLE. They occur
due to formation of antibodies against blood cells and represent a form
of type II hypersensitivity reaction. Anemia in SLE is caused by
autoimmune hemolysis, and develops due to formation of warm IgG
antibodies to RBCs. Itis characterized by spherocytosis, a positive
direct Coombs test, and extravascular hemolysis. The pathogenesis of
SLE-associated thrombocytopenia is identical to that of ITP; antibodies
against platelets are formed causing destruction of platelets.
Neutropenia due to antibody-mediated destruction of WBCs also occurs,
but this is less common.
(Choice A) Bone marrow hypoplasia or aplasia occurs due to exposure to
environmental toxins, certain medications (antimetabolites,
chloramphenicol), infectious agents (parvovirus) or as an inherited
condition (Fanconi anemia).
(Choice B) Ineffective hematopoiesis refers to blood cell breakdown in
bone marrow before release into the circulation. Thalassemias and
myelodysplastic syndrome are the examples of such conditions.
(Choice C) Hypersplenism can result in abnormal pooling of blood cells
in the spleen with subsequent destruction of these cells. It manifests
with pancytopenia and splenomegaly, and may occur in cirrhosis,
malaria, sickle cell disease and many other disorders.
(Choice E) Dilutional pancytopenia can occur after a massive packed red
blood cell transfusion or massive infusions of crystalloid solutions
due to increased plasma volume out of proportion to the number of blood
cells.

Educational Objective:
Pancytopenia (decreased RBC, WBC and platelets) is common in patients
with SLE. It occurs due to the formation of autoantibodies against
blood cells, a form of type II hypersensitivity reaction.

__________________________________________________________ 9
USMLE WORLD STEP 2 CK (INTERNAL MEDICINE) *Block
1*
Q NO 9: A 52-year-old male presents to his primary care physician
complaining of dark urine and feeling very itchy all over. Further
inquiry reveals that he also has been experiencing episodes of right
upper quadrant pain that awakens him from sleep and lasts for 2-4 hours
at a time. He says that the pain radiates to his right scapula and is
severe, dull, and constant. His medical history is significant for
hypothyroidism managed with medication and a remote cholecystectomy. He
has smoked half a pack of cigarettes per day since high school and
drinks alcohol on social occasions. He does not use illicit drugs.
Laboratory evaluation includes the following:
Liver studies
Total bilirubin 5.2 mg/dL
Direct bilirubin 4.1 mg/dL
Alkaline phosphatase 455 U/L
Aspartate aminotransferase (ASTI SGOT) 80 U/L
Alanine aminotransferase (ALT, SGPT) 101 U/L
A sphincter of Oddi spasm is suspected based on his symptoms and
history. An endoscopic retrograde cholangiopancreatography (ERCP) with
sphincterotomy is performed. Two days later, the patients bilirubin
begins to decrease, but imaging reveals air within in the biliary tree.
What is the most likely cause of this finding?

A. Normal finding after ERCP


B. Acute pancreatitis
C. Biliary infection with gas-forming bacteria
D. Gangrenous cholecystitis
E. Biliary enteric fistula

Explanation:
Endoscopic retrograde cholangiopancreatography(ERCP) is one of the most
sensitive and specific tools for imaging the biliary system, and is
particularly useful if therapeutic intervention is planned. It is
typically used to explore the common bile duct, to clear stones from
the common bile duct, to sample tissue for diagnostic purposes, to
relieve sphincter of Oddi dysfunction by incising the musculature, and
to palliate biliary obstruction when surgery is not planned. However,
the procedure is not without risk with an overall complication rate of
5-10% and a mortality rate of 1.5%. Some of the more common
complications of ERCP include pancreatitis, perforation (resulting in
biliary enteric fistula, especially after sphincterotomy), biliary
peritonitis, sepsis, hemorrhage, and adverse effects from the contrast.
sedative, or anticholinergic agents. Given this patients presentation,
then, it is most likely that he has a biliary enteric fistula secondary
to perforation during the ERCP procedure. Surgical correction of the
condition is indicated.
(Choice A) Certain procedures (e.g., the air insufflation test for
infertility) can result in the finding of free air under the diaphragm,
but this is not the case with ERCP.
(Choice B) Acute pancreatitis is a known complication of ERCP but will
not cause the appearance of gas in the biliary tree. Pancreatitis
occurs in approximately 5% of all patients undergoing ERCP and is seen
in up to 25% of patients who are undergoing ERCP for sphincter of Oddi
dysfunction specifically.
(Choice C) Biliary infection with gas-forming bacteria will create
shadowing within the biliary tree and will cause the patient to look

10 __________________________________________________________
USMLE WORLD STEP 2 CK (INTERNAL MEDICINE) *Block
1*
very toxic. Because the patient in this scenario underwent ERCP two
days ago, however, an iatrogenic cause is much more likely.
(Choice D) Gangrenous cholecystitis will cause the formation of gas
within the gallbladder, not the biliary tree. In addition, patients
with this condition are septic.

Educational Objective:
One of the known complications of ERCP is an iatrogenic biliary enteric
fistula characterized by the presence of air in the biliary tree. Other
complications include pancreatitis, biliary peritonitis, sepsis,
hemorrhage, and adverse effects from the contrast, sedative, or anti
cholinergic agents.

__________________________________________________________ 11
USMLE WORLD STEP 2 CK (INTERNAL MEDICINE) *Block
1*
Q NO 10: A 45-year-old recently migrated Mexican farmer comes to your
office because of dyspnea and fatigue, since last 2 months. His vital signs
are, BP: 126/80mm Hg, PR: 80/min, RR: 16/min and Temperature: 37 C
(98 F). On examination, he has pedal edema, elevated jugular venous
pressure with positive Kussmauls sign and increased abdominal girth with
free fluid. Auscultation reveals reduced intensity of apex beat with an
early heart sound following S2. Jugular venous pressure tracing shows
prominent x and y descent. Which of the following is the most likely
cause for this patients symptoms?

A. Cor pulmonale
B. Tuberculosis
C. Viral infection
D. Psittacosis
E. Pneumoconiosis

Explanation:
The diagnosis in this patient is constrictive pericarditis.
Constrictive pericarditis results from obliteration of pericardial
space and fibrosis of pericardium following an acute fibrinous or
serofibrinous pericarditis or chronic pericardial effusion.
Tuberculosis is a very common cause of constrictive pericarditis,
especially in developing countries. This patient is a recently migrated
Mexican farmer, which puts TB on the top of the list of differentials
for his constrictive pericarditis (Option E). Some other causes of
constrictive pericarditis are idiopathic (42% in USA), post
radiotherapy (31% in USA), post surgical (11%), connective tissue
disorders, neoplasm, uremia, sarcoidosis, etc.
Constrictive pericarditis will lead to inability of ventricle to fill
properly during diastole and would further cause the signs and symptoms
of decreased cardiac output (fatigue muscle wasting etc) and/or signs
and symptoms of venous overload like elevated JVP, dyspnea, ascites,
positive Kussmauls sign, pedal edema, tender hepatomegaly etc. Sharp
x and y descent on central venous tracing is characteristic of
constrictive pericarditis as is the presence of pericardial knock
(early heart sound heard after S2).
Option D and E: The clinical presentation of this patient is highly
suggestive of constrictive pericarditis and psittacosis and
pneumoconiosis are not an etiological cause of this condition.
Option A: This patient has no finding suggestive of chronic lung
disease, which would lead to cor pulmonale. Also cor pulmonale would
not cause constrictive pericarditis.

Educational objective:
Tuberculosis is the most common cause of constrictive pericarditis, in
immigrant population. It should be considered in patients with
unexplained elevation of JVP and history of predisposing condition.

12 __________________________________________________________
USMLE WORLD STEP 2 CK (INTERNAL MEDICINE) *Block
1*
Q NO 11: A 42-year-old male presents to your office complaining of back
pain that started two days ago after carrying heavy packages. He denies
any weakness or sensory changes in his legs. His past medical history is
insignificant. He is not taking any medications and denies drug abuse. His
temperature is 36.7 C (98 F), blood pressure is 120/76 mmHg, pulse is
80/min, and respirations are 16/min. Physical examination reveals
paravertebral tenderness. Lower extremity power is 5/5 and the deep
tendon reflexes are 2+. Babinskis sign is negative. Straight-leg raising
test is negative at 90 degrees. What is the most probable diagnosis in this
patient?

A. Multiple myeloma
B. Ankylosing spondylitis
C. Compression fracture of the vertebrae
D. Lumbosacral strain
E. Herniated disk
F. Metastatic tumor
G. Increased lumbar lordosis
H. Spondylolisthesis
I. Epidural abscess
J. Spinal stenosis
K. Abdominal aortic aneurysm
L. Cauda equina syndrome
M. Transverse myelitis
N. Vertebral body osteomyelitis

Explanation:
Lumbosacral strain is the most common cause of back pain. Itis
estimated that the lifetime risk of lumbosacral strain is close to 8O%.
The clinical scenario described is typical. The pain starts acutely
after physical exertion, and it is concentrated in the lumbar area,
usually without radiation to the thighs. Physical examination reveals
local tenderness and contraction of the paraspinal muscles. A straight-
leg raising test and neurologic examination are typically normal. The
treatment includes NSAIDs and early mobilization.
A herniated disk (Choice E) is characterized by acute pain that
radiates to the thighs and typically below the knee. Straight-leg
raising test is positive.
A compression fracture of the vertebrae (Choice C) presents as acute
intense pain, and local spinal tenderness is usually observed.
Predisposing factors are usually obvious (postmenopausal or senile
osteoporosis, steroid treatment).
Ankylosing spondylitis (Choice B) and multiple myeloma (Choice A) are
characterized by chronic back pain. A metastatic or primary tumor
(Choice F) is not likely in this case.

Educational Objective:
Lumbosacral strain is the most common cause of acute back pain. The
typical clinical scenario includes acute onset of the back pain after
physical exertion, absence of radiation, presence of paravertebral
tenderness, negative straight-leg raising test, and normal neurologic
examination.

__________________________________________________________ 13
USMLE WORLD STEP 2 CK (INTERNAL MEDICINE) *Block
1*
Q NO 12: A 43-year-old male presents to a physician with an ulcer on the
shaft of his penis. The ulcer is non-tender, with a raised border and a
smooth base. There is bilateral inguinal adenopathy. The rest of the
examination is unremarkable. Dark field microscopy of a specimen from
the ulcer base reveals spirochetes. Which of the following additional
screening studies should be performed on this patient?

A. VDRL
B. FTA-ABS
C. HIV antibodies by ELISA
D. Proctosigmoidoscopy
E. Serum prostate specific antigen

Explanation:
This patients clinical picture is consistent with primary syphilis.
Darkfield microscopy is especially useful in diagnosing primary
syphilis, and visualization of the spirochetes (as in this case)
confirms the diagnosis. This patients syphilis infection suggests that
he may be involved in high-risk sexual activity, also putting him at
risk for HIV exposure. After proper counseling, HIV screening using
ELISA should be offered.
(Choices A and B) VDRL and ETA-ABS both test for syphilis exposure. In
this case, the diagnosis has already been confirmed by darkfield
microscopy, so there is no need for serological testing.
(Choices D and E) Proctosigmoidoscopy is a screening test for
colorectal cancer, and serum prostate specific antigen is a screening
test for prostate cancer. Screening with both tests is usually
instituted at age 50.

Educational Objective:
Know the screening and diagnostic tests for syphilis. Know how to
screen high-risk patients for sexually transmitted diseases.

14 __________________________________________________________
USMLE WORLD STEP 2 CK (INTERNAL MEDICINE) *Block
1*
Q NO 13: A 73-year-old man presents to the emergency department
complaining of lower abdominal pain and nausea. He denies any
vomiting or diarrhea, and his last bowel movement was two days
ago. The patient also notes that several days ago he began taking
amitriptyline for chronic neck pain. He does not smoke or consume
alcohol. On physical examination, his blood pressure is 160/70 mmHg
and his heart rate is 100/min. His lung fields are clear to
auscultation. Palpation of the abdomen reveals fullness and
tenderness along the midline below the umbilicus. Which of the
following is the best initial management for this patient?
A. Abdominal CT scan
B. Upright abdominal x-ray
C. Barium enema
D. Broad spectrum antibiotics
E. Urinary catheterization
F. IV fluids, analgesics, and observation

Explanation:
This patient presents with abdominal pain and suprapubic fullness several
days after starting amitriptyline for chronic pain. This presentation is
consistent with amitriptyline-induced urinary retention. Amitriptyline is a
tricyclic antidepressant with anticholinergic properties. Because both the
detrusor muscle and urethral sphincter are under muscarinic control,
anticholinergic agents will reduce detrusor contraction and prevent
urethral sphincter relaxation. The result is urinary retention.
Urinary catheterization would serve two purposes in this case. First, it
can document a postvoid residual bladder volume of greater than 50 ml,
which is considered diagnostic of urinary retention. Plus, catheterization
will provide symptomatic relief as it drains urine from the bladder. The
patient should also discontinue amitriptyline therapy.
(Choice A) An abdominal CT scan would reveal a distended bladder in this
patient and may also show hydronephrosis. However, CT scans are much more
expensive and time consuming than urinary catheterization and will not
provide symptomatic relief.
(Choice B) An upright abdominal x-ray may show a distended bladder but it
is nota reliable test for evaluating urinary retention. Abdominal x-rays
are especially helpful for diagnosing ileus or small bowel obstruction.
(Choice C) Barium enemas are used to diagnose luminal abnormalities of the
colon, like colon cancer or diverticulosis.
(Choice D) Broad-spectrum antibiotics are appropriate when a urinary tract
infection with urinary retention is suspected. This patient does not
complain of fever, chills, or dysuria, making infection less likely.
(Choice F) IV fluids, analgesics, and observation are the treatment for
nephrolithiasis, or kidney stones. Patients with kidney stones typically
present with intense flank pain and hematuria, not suprapubic fullness. IV
fluids would actually exacerbate this particular patients symptoms.

Educational Objective:
Drugs with anti cholinergic properties can cause acute urinary retention by
preventing detrusor muscle contraction and urinary sphincter relaxation.
The treatment is medication discontinuation and urinary catheterization.

__________________________________________________________ 15
USMLE WORLD STEP 2 CK (INTERNAL MEDICINE) *Block
1*
Q NO 14: A 65-year-old man presents with sudden onset of severe right
knee pain with fever and chills. He is a retired soccer player with history
of osteoarthritis involving both knees. On further questioning, he denies
having any other symptoms. Examination of his knee joint reveals
redness warmth, tenderness with effusion and marked limitation in range
of movements. His vital signs are pulse: 86/min, BP: 140/90mm Hg, PR:
18/min, and Temp: 38.5 C (102 F). Which of the following white cell
counts in joint fluid aspirate best correlates with the above presentation?

A. 35000/uL
B. 90000/uL
C. 150/uL
D. 55000/uL
E. 45000/uL

Explanation:
Explanation: The sudden onset of acute monoarticular arthritis in a
previously damaged joint with no other associated symptoms other than
chills and fever suggests septic arthritis. The joint involved will be
tender and erythematous with large effusion. It is most commonly caused
by staphylococcus aureus. In intravenous drug abusers and
immunocompromised patients, gram-negative bacteria such as E. Coli and
pseudomonas aeruginosa are commonly isolated. In septic arthritis the
leukocyte count of synovial fluid exceeds 50000 and often 100 000/uL.
Systemic antibiotic therapy when started promptly results in full
recovers.
Choice A: Cell counts in the range of 2000/uL to 50000/uL suggest mild
to moderate inflammation as seen in rheumatoid and crystalline
arthritis.
Choice C: Cell counts less then 200 are normal.
Choice D 55000/uL and Choice E 45000/uL cell counts suggest gout or
moderate inflammation.

Educational Objective:
Acute inflammatory monoarticular arthritis in a previously damaged
joint suggests septic arthritis. Leukocyte counts in synovial fluid
exceeding 50.000 or even 100,000/ul should make you think of septic
arthritis.

16 __________________________________________________________
USMLE WORLD STEP 2 CK (INTERNAL MEDICINE) *Block
1*
Q NO 15: A 21-year-old previously healthy man presents to your office for
a routine check-up. He has no current complaints. He does not smoke or
consume alcohol. His family history is significant for cystic fibrosis in his
older brother. He is sexually active with one partner and uses condoms
regularly. He visits a dentist twice per year. His temperature is 36.6C (97.9
F), pulse is 78/min, respirations are 14/min, and blood pressure is 120/76
mmHg. Physical examination reveals several non-tender, rubbery cervical
lymph nodes, each measuring approximately 1 cm in diameter. There is no
hepatosplenomegaly. Which of the following is the best next step in
management of this patient?

A. Lymph node biopsy


B. Dicloxacillin
C. Prednisone
D. Erythromycin
E. Acyclovir
F. Observation

Explanation:
This patient has asymptomatic localized lymphadenopathy. There are multiple
causes of cervical lymphadenopathy, but it most commonly develops in the
setting of an upper respiratory infection. The physical exam often helps to
distinguish between benign etiologies and more serious ones. Small,
rubbers lymph nodes are rarely pathologic and are often found in healthy
children and young adults. Nodes less than 1 .0 cm in diameter are almost
always due to a benign process. In contrast, nodes associated with cancer
tend to be firm and immobile. A nodal diameter> 2.0 cm is associated with a
greater likelihood of malignancy or granulomatous disease. At this point,
this patients lymphadenopathy can be observed. Should the patient develop
symptoms (e.g., fever, chills, or weight loss) or if the nodes grow, he
should receive further evaluation. Hard cervical nodes in an older patient
or smoker would prompt an investigation for metastatic cancer or
oropharyngeal neoplasia.
(Choice A) Biopsy may be required if the lymph node diameter is greater
than 2.0 cm.
(Choice B) Dicloxacillin would be an acceptable choice if the patient had
fluctuant cervical lymph nodes, a condition typically caused by
staphylococcal or streptococcal infection. However, it is not necessary to
treat a patient with asymptomatic cervical lymphadenopathy with an
antibiotic.
(Choice C) Sarcoidosis can cause cervical lymphadenopathy and often
responds promptly to corticosteroid therapy. This patient does not have
symptoms or generalized lymphadenopathy suggestive of a systemic
inflammatory condition.
(Choice D) Erythromycin is a macrolide antibiotic that is among the first-
line agents for treating streptococcal pharyngitis and mild community
acquired pneumonia.
(Choice E) Herpes simplexvirus (HSV) and varicella zoster virus (VZV)
infections are treated with acyclovir. This patient does not have skin
manifestations or other symptoms to suggest infection with one of these
viruses.
Educational Objective:
Small cervical lymph nodes are a common observation in children and young
adults. Patients with asymptomatic, soft lymph nodes can be observed for
node growth or the development of symptoms.

__________________________________________________________ 17
USMLE WORLD STEP 2 CK (INTERNAL MEDICINE) *Block
1*
Q NO 16: A 47-year-old woman loses consciousness for 2 minutes while
shopping in a supermarket. In the emergency room, she recounts feeling
nausea and warmth spreading over her body immediately before passing
out. She has never had a similar episode before. She has not seen a
doctor for several years and does not take any medications nor does she
use tobacco, alcohol or drugs. Her family history is unremarkable. Which
of the following most likely caused this episode?

A. Cardiac arrythmia
B. Seizure
C. Neurocardiogenic syncope
D. Heart valve disease
E. Orthostatic hypotension
F. Transient ischemic attack

Explanation:
Neurocardiogenic syncope, also known as vasovagal syncope, is a common
cause of fainting. It occurs due to excessive vagal tone and is
characterized by nausea, diaphoresis, tachycardia, and pallor
immediately prior to the syncopal episode. Lt frequently occurs in
response to stress, pain, and certain bodily actions (e.g. urination).
It is particularly common in young women. Vasovagal syncope can be
diagnosed with the tilt table test.
(Choice A) Conduction disorders (e.g. AV block), tachyarrhythmias (e.g.
ventricular or supraventricular tachycardia), and disorders of
automaticity (e.g. sick sinus syndrome) can all cause fainting.
Generally there are no preceding signs or symptoms, except for
palpitations in some. Patients usually have underlying cardiac disease.
(Choice B) Seizures and syncopal episodes are sometimes difficult to
distinguish. This patient does not have clear signs of seizure (e.g.
tongue biting), a history of seizures, or a post-ictal state, making
syncope more likely.
(Choice D) Syncope due to aortic stenosis most often occurs with
activity. Dyspnea, chest pain, and fatigue on exertion are symptoms of
aortic stenosis that generally occur earlier in the disease course.
(Choice E) Orthostatic hypotension is defined as a drop in systolic
blood pressure greater than 20 mm Hg after the patient rises to a
standing position. It is most common in the elderly, diabetics, those
with autonomic neuropathy (e.g. Parkinsons disease), those with
hypovolemia, and people taking diuretics, vasodilators, or adrenergic-
blocking agents. Prolonged recumbence increases the risk. Patients
commonly experience pre-syncopal lightheadedness.
(Choice F) Transient ischemic attacks (TIAs) can produce syncope, but
are a rare cause because the TIA must affect the posterior circulation
and brain stem in order for syncope to occur. This patient has no risk
factors for stroke.

Educational Objective:
Neurocardiogenic, or vasovagal, syncope occurs due to excessive vagal
tone. Episodes are preceded by nausea, diaphoresis, tachycardia, and
pallor. Pain, stress, and situations like medical needles and urination
can all precipitate vasovagal syncope.

18 __________________________________________________________
USMLE WORLD STEP 2 CK (INTERNAL MEDICINE) *Block
1*
Q NO 17: A 32-year-old woman presents to her primary care physician
complaining of vomiting and very dark yellow urine that began four
days ago while she was at work. She adds that two of her siblings have
similar symptoms. Three weeks ago, her family returned from a trip to
India. Prior to this, the woman was healthy and had no medical
conditions that required treatment. She has no known drug allergies.
Physical examination reveals scleral icterus and right upper quadrant
tenderness with hepatomegaly. Serological testing confirms the
diagnosis of hepatitis E infection. Which of the following is true about
hepatitis F?

A. Hepatitis E is primarily sexually transmitted


B. Hepatitis E can be effectively prevented by vaccination
C. Hepatitis F has a very high rate of progression to fulminant
hepatitis in the pregnant female
D. Hepatitis F infection can evolve into a carrier state
E. Hepatitis F is endemic in the United States

Explanation:
Hepatitis E virus (HEV) is an RNA virus that causes an illness similar to
that seen with hepatitis A infection. Histologically, hepatic biopsy in
this patient population reveals focal necrosis, ballooned hepatocytes, and
acidophilic hepatocytic degeneration. Transmission of HEV typically occurs
through fecally contaminated water in endemic areas with person-to-person
transmission rare. Hepatitis E virus has a predictable geographical
distribution, and is most commonly found in India, Asia, Africa, and
Central America. It is highly unusual in the United States. Individuals
aged 15-40 years are most commonly infected and appear to have an
incubation period of 15-60 days, with abrupt onset of symptoms. Jaundice,
malaise, anorexia, nausea, vomiting, abdominal pain, fever, and
hepatomegaly are the more common signs and symptoms. Laboratory findings
include elevated serum bilirubin, ALT, and AST. Diagnosis is confirmed with
detection of HEV RNA by PCR in the serum or feces or by the detection of
1gM antibodies to HEV.
Hepatitis E infection is self-limited, with no chronic carrier state. There
is no progression to chronic hepatitis, cirrhosis, or hepatocellular
carcinoma. However, fulminant hepatitis can occur, and results in an
overall case fatality rate of 0.5- 3%. More alarmingly, fulminant hepatitis
occurs in 15-25% of pregnant women infected with hepatitis E virus,
especially those in the third trimester. There is also evidence to suggest
that hepatitis E infection can be vertically transmitted, with significant
perinatal morbidity and mortality.
There is little information available about the efficacy of immune globulin
in HEV prevention, and therefore it is not typically recommended. Travelers
to endemic areas should be advised to minimize exposure to water of unknown
purity.
(Choice A) Hepatitis E is primarily transmitted via the fecal-oral route,
not sexual contact.
(Choice B) An effective HEV vaccine is not widely available at this time.
(Choice D) Hepatitis E virus infection does not result in a chronic carrier
state.
(Choice E) Hepatitis E is very rare in the United States and is more
commonly seen in India, Asia, Africa, and Central America.

Educational Objective:

__________________________________________________________ 19
USMLE WORLD STEP 2 CK (INTERNAL MEDICINE) *Block
1*
Infection with hepatitis E virus has a high rate of progression to
fulminant hepatitis in pregnant women, especially in the third trimester.

20 __________________________________________________________
USMLE WORLD STEP 2 CK (INTERNAL MEDICINE) *Block
1*
Q NO 18: A 66-year-old white male presents to your office with a five
month history of difficulty walking. He experiences right thigh pain
while walking fast or climbing one flight of stairs. Pain is relieved with
rest. His past medical history is significant for hypercholesterolemia
controlled with simvastatin. He smokes two packs a day and
consumes alcohol occasionally. His blood pressure is 155/95 mmHg
and heart rate is 70/min. Physical examination reveals systolic
murmur over the right second intercostal space, right carotid bruit,
diminished pulsation over the right popliteal artery, and absent
dorsalis pulsations in both legs. Which of the following medications is
the best choice to control this patients hypertension?
A. Hydrochlorothiazide
B. Metoprolol
C. Amlodipine
D. Enalapril
E. Doxazosin

Explanation:
This patient presents with intermittent claudication significantly
restricting his daily activities and probable asymptomatic carotid
artery and aortic stenosis. An unfavorable lipid profile and smoking
are other risk factors. Treatment of hypertension is very important,
because this patient belongs to a high-risk group for future
cardiovascular events. The best initial choice for the treatment of
hypertension in this patient seems to be a dihydropyridine calcium
channel blocker like amlodipine. Calcium channel blockers have good
peripheral vasodilating properties that can help to diminish the
symptoms of intermittent claudication. They are also metabolically
neutral, not affecting plasma lipid profile.
Thiazide diuretics, like hydrochlorothiazide (Choice A), are not
metabolically neutral and do not affect intermittent claudication:
therefore, a calcium channel blocker is a better choice for this
patient.
Metoprolol, a beta-blocker (Choice B), can worsen the symptoms of
peripheral vascular disease.
Beta-blockers should be stopped if the patient develops significant
peripheral vascular disease.
Enalapril, an ACE inhibitor (Choice D), can be considered, but it would
not relieve the symptoms of intermittent claudication.
Doxazosin (Choice E) is an alpha-blocker with a favorable metabolic
profile. It is not considered as a first-line drug in the treatment of
hypertension, because the long-term effects of alpha-blocker therapy
are not clear. Moreover, some side effects of the alpha-blocker therapy
may be disturbing (e.g., orthostatic hypotension).

Educational Objective:
In patients presenting with significant peripheral vascular disease,
calcium channel blockers are preferred as antihypertensive agents.

__________________________________________________________ 21
USMLE WORLD STEP 2 CK (INTERNAL MEDICINE) *Block
1*
Q NO 19: Which of the following patient will have the lowest rate of A.
progression of chronic hepatitis C to hepatic fibrosis and cirrhosis? A
non-alcoholic female who acquired hepatitis C at age of 18
B. An occasional alcoholic male who acquired hepatitis C at 44 years
age
C. A non-alcoholic male co infected with HBV and HCV
D. A non-alcoholic female cc infected with HIV and HCV
E. A non-alcoholic male who acquired hepatitis at age of 18

Explanation:
About 50 to 70% of patients with acute hepatitis C progress to chronic
hepatitis C regardless of the mode of acquisition. Progression of liver
disease in case of chronic hepatitis C is best assessed by liver
histology.
Progression of liver disease in patients with chronic hepatitis is
relatively more rapid in following conditions:
1. Male sex
2. Acquiring infection after age of 40
3. Longer duration of infection
4. Co infection with HBC or HIV
5. Immunosuppression
6. Liver co morbidities like alcoholic liver disease, hemochromatosis,
alpha-i anti trypsin deficiency
Factors independently associated with high rates of liver fibrosis in
chronic hepatitis C patients are:
1. Male gender
2. Acquiring infection after age of 40
3. Alcohol intake: alcohol intake in any amount can hasten the
progression of fibrosis in patients with chronic hepatitis C
(Choice A) She has the least progression of fibrosis among the given
sets of patients due to her female sex, earlier age of acquisition of
infection, non alcoholic state, and no liver co morbidities or
infections.
(Choice B) Infection after the age of 40 is an independent risk factor
for progression of fibrosis.
(Choices C and D) Co infection with hepatitis B or HIV is a risk factor
for rapid progression of fibrosis.
(Choice E) Although patient has infection at younger age and is a non-
alcoholic, his male sex is an additional risk factor as compared to
patient in Choice A.

Educational Objective:
Know the risk factors for rapid progression of liver fibrosis in
patients with chronic hepatitis C.

22 __________________________________________________________
USMLE WORLD STEP 2 CK (INTERNAL MEDICINE) *Block
1*
Q NO 20: A 65-year-old man presents with complaints of decreased vision
in both eyes. His visual impairment has been progressively worsening over
the past five months. He was diagnosed with diabetes ten years ago. His
current medications are metformin and glyburide. His blood pressure is
140/90 mm Hg, pulse is 82/min, respirations are 14/min, and temperature
is 98.4 F (36.88 C). Examination shows decreased visual acuity in both
eyes. Ophthalmoscopy reveals microaneurysms, dot and blot
hemorrhages, hard exudates, and macular edema. Which of the following
is the most likely diagnosis?

A. Central retinal vein occlusion


B. Diabetic retinopathy
C. Macular degeneration
D. Retinal detachment
E. Open angle glaucoma

Explanation:
Diabetic retinopathy is the leading cause of blindness in the USA. It
occurs in both insulin dependent and non-insulin dependent diabetes
mellitus. There are 3 main categories:
1. background or simple retinopathy- consists of microaneurysms,
hemorrhages, exudates, and retinal edema. as in this patient
2. pre-proliferative retinopathy- with cotton wool spots
3. proliferative or malignant retinopathy - consists of newly formed
vessels.
Patients are usually asymptomatic at first, despite early signs of
retinopathy (e.g., microaneurysms). Visual impairment occurs with the
development of macular edema. Argon laser photocoagulation is the
suggested treatment for the prevention of complications.
(Choice A) Central retinal vein occlusion is characterized by sudden,
unilateral visual impairment that is usually noted upon waking in the
morning. Diabetics are at increased risk for retinal vein occlusion.
Ophthalmoscopy reveals disc swelling, venous dilation and tortuosity,
retinal hemorrhages and cotton wool spots.
(Choice C) Macular degeneration affects central vision. It is
characterized by distorted vision and central scotoma. Cigarette
smoking increases the risk of macular degeneration. Ophthalmoscopy
findings vary according to the type (i.e., atrophic vs. exudative). The
atrophic form is characterized by multiple sores in the macular region,
while the exudative form is characterized by new blood vessels that may
leak, bleed, and scar the retina.
(Choice D) Retinal detachment occurs unilaterally. It presents as
blurred vision that progressively worsens. Ophthalmoscopy reveals the
retina hanging in the vitreous.
(Choice E) Open angle glaucoma is also seen in diabetics, but is
characterized by gradual loss of peripheral vision, resulting in tunnel
vision. Ophthalmoscopy shows pathologic cupping of the optic disc.

Educational Objective: The 3 main categories of diabetic retinopathy


are background or simple (microaneurysms, hemorrhages, exudates,
retinal edema), pre-proliferative (cotton wool spots), and
proliferative or malignant (neovascularization). Visual impairment

__________________________________________________________ 23
USMLE WORLD STEP 2 CK (INTERNAL MEDICINE) *Block
1*
occurs with the development of macular edema. Argon laser
photocoagulation is performed for the prevention of complications.

24 __________________________________________________________
USMLE WORLD STEP 2 CK (INTERNAL MEDICINE) *Block
1*
Q NO 21: A 25-year-old white male presented with polyarthralgias, and
severe heel pain. He discloses that he had a sexual intercourse with
prostitute: since then he started having urethral discharge. On
examination all his digits are diffusely swollen and appears like
sausage digits. Gram stain of the urethral discharge and joint
aspiration is negative for organisms. Which of the following is the most
appropriate next step in the management of this patient?

A. Treatment with corticosteroids


B. Treatment with NSAIDs
C. Treatment with ceftriaxone IM one dose
D. Draw blood cultures
E. Treatment with methotrexate

Explanation:
Explanation: This patient is most likely suffering from arthritis
reactive to genitourinary infection by Chlamydia. Reactive arthritis is
a form of seronegative spondyloarthropathy. This patient also has
evidence of enthesopathy that has caused heel pain and sausage digits
in this patient. Enthesopathy is quite specific for
spondyloarthropathy.
The treatment of choice for reactive arthritis or Reiters syndrome is
with NSAIDs. Tetracycline may be added to them if the disease is due to
genitourinary infection with Chlamydia and this will result in early
resolution of symptoms due to anti-chlamydial and anti-inflammatory
activity of tetracycline. Patients who are refractory to the above
treatment may benefit from sulfasalazine, infliximab and methotrexate.
Treatment with IM ceftriaxone is indicated when there is gonococcal
infection that is a gram-negative diplococcus. Gonococcal infection is
not a cause of reactive arthritis. In cases of gonococcal infection,
gram stain of the urethral discharge will shows gram-negative
diplococcus.
Blood cultures are not the appropriate choice in this setting since
this is not a septic process.

Educational Objective:
NSAIDs are the mainstay of treatment of Reiters syndrome/reactive
arthritis.

__________________________________________________________ 25
USMLE WORLD STEP 2 CK (INTERNAL MEDICINE) *Block
1*
Q NO 22: Ms. Sally, a 54-year-old woman, comes to the office with
complaints of increased weight in the last 4 months. She denies any
change in appetite or eating habits, but has gained 50 lb (27.6kg).
She is on no medications and is only social drinker. She is yew diet
conscious and follows the exact instructions in Dr. At kins Diet
revolution. She smokes 1 pack/day cigarettes for the past l6years.
Physical examination shows an obese abdomen and thin legs, but is
otherwise unremarkable. Further work-up confirms the diagnosis of
small cell carcinoma of lungs. Which of the following set of tests will
be most related to patients presenting symptoms?
What will be the levels of:
Cortisol, ACTH levels, Low Dose Dexamethasone Suppression,
High Dose Dexamethasone Suppression

A. Increased, Increased, No Suppression, Suppression


B. Increased, Decreased No Suppression No Suppression
C. Increased, Increased, Suppression, No Suppression
D. Increased, Increased, No Suppression No Suppression
E. Normal, Normal, Suppression, Suppression

Explanation:
Some benign or malignant (cancerous) tumors that arise outside the
pituitary can produce ACTH. This condition is known as Ectopic ACTH
syndrome. Lung tumors cause over 50% of these cases. Men are affected 3
times more frequently than women. The most common forms of ACTH-
producing tumors are oat cell, or small cell lung cancer, which
accounts for about 25 % of all lung cancer cases, and carcinoid tumors.
Other less common types of tumors that can produce ACTH are thymomas,
pancreatic islet cell tumors, and medullary carcinomas of the thyroid.
Cushings syndrome work up shows the results as shown in Choice D.
Choice A is seen in pituitary adenoma while Choice B is seen in Adrenal
neoplasia. Choice C is a distracter while Choice E is normal.
Since, it is an ectopic ACTH syndrome, ACTH and cortisol will be
elevated. Because the amount of ACTH is very high, itis difficult to
suppress even with high dose dexamethasone.

Educational Objective:
Patients with ectopic producation of ACTH have very high ACTH levels
and their cortisol level is not suppressed with high dose dexamethasone
suppression test.

26 __________________________________________________________
USMLE WORLD STEP 2 CK (INTERNAL MEDICINE) *Block
1*
Q NO 23: Ten measurements of systolic blood pressure were obtained from
a patient in the ICU using an intra-arterial cannula over several hours. The
maximal value recorded is 120 mmHg, and the minimal value is 100 mmHg.
If the next measurement shows the value of 240 mmHg, which of the
following is most likely to happen?

A. The mean would increase significantly


B. The median would increase significantly
C. The mode would increase significantly
D. The standard deviation would not change
E. The range would not change

Explanation:
An outlier is defined as an extreme and unusual value observed in a
dataset. It may be the result of a recording error. a measurement
error, or a natural phenomenon. An outlier can affect the measures of
central tendency, as well as the measures of dispersion. For example,
the mean is extremely sensitive to outliers and easily shifts toward
them. In this case, the value of 240 mmHg is the outlier.
(Choice B) The median is much more resistant to outliers, because itis
located in the middle of the dataset where the observations usually do
not differ much from each other.
(Choice C) The mode is not affected by outliers, because they do not
change the most frequent value observed.
(Choice D) The standard deviation is sensitive to outliers, because it
is the measure of dispersion within the dataset, and outliers
significantly increase the dispersion.
(Choice E) The range is equal to the maximal value minus the minimal
value; therefore, it would definitely change.

Educational Objective:
An outlier is defined as an extreme and unusual observed in a dataset.
The mean is very sensitive to outliers and easily shifts toward them.
The median and mode are more resistant to outliers.

__________________________________________________________ 27
USMLE WORLD STEP 2 CK (INTERNAL MEDICINE) *Block
1*
Q NO 24: A 64-year-old male presents to you with complaints of a cough,
visual changes, weight loss and pain in his right arm. He says that he has
not been feeling well for the past 2 months. He is a smoker and diabetic.
He denies any drug allergies. Examination is unremarkable. ECG reveals
normal sinus rhythm. The MRI is shown below. What is the most likely
pathology responsible for the individuals symptoms?

A. Aspergilloma
B. Carcinoid
C. Pancoast tumor
D. Pneumonia
E. Lung abscess

Explanation:
The patient has all the presentations of a malignancy with pain in his
right arm. The MRI shows a mass in the apex of the lung. This is
classic bra Pancoast tumor. Other findings may be the presence of
Homers syndrome (miosis, anhydrosis and ptosis). Pancoast tumors
usually grow in the apex of the lung (superior sulcus) and tend to
invade the upper chest wall, ribs, sympathetic chain and lower brachial
plexus. When the tumor invades the lower brachial plexus, it can cause
severe pain in the arm along the ulnar nerve. Preoperative radiation is
administered before resection of this tumor. Pancoast tumors maybe of
any histology but the majority are either squamous cell or
adenocarcinomas (Choice C).
(Choice A) Aspergilloma can grow in the apex of the lung but does not
cause arm pain. The patent may present with hemoptysis, fever, weight
loss and a chronic cough. The mass is usually mobile and moves with
position. The diagnosis is made by finding of hyphae in the specimen.
The occasional aspergilloma can spontaneously regress.
(Choice B) Carcinoid tumors usually do not grow in the lung apex. These
tumors are usually centrally located and easily visible with a
bronchoscope. They may present with a cough and recurrent hemoptysis.
The tumors are slow growing and have the best prognosis of all lung
cancers. Carcinoid syndrome is a rare presentation.
(Choice D) Pneumonia presents as a consolidation but does not cause arm
pain. The patient with a large pneumonia can present with cough,
copious secretions, fever, elevated WBC, and weight loss. Diagnosis can

28 __________________________________________________________
USMLE WORLD STEP 2 CK (INTERNAL MEDICINE) *Block
1*
be made by gram stain of sputum specimens. An untreated pneumonia can
lead to a parapneumonic effusion and empyema.
(Choice E) Lung abscess can present as above but the patient is usually
sick and ill. Fevers, chills, oral secretions, weight loss, chest pain
and elevated WBC are common findings. An air fluid level is diagnostic.
Arm pain does not occur with lung abscess. Diagnosis is made by
clinical presentation, radiological features and cultures of oral
secretions. Treatment is prolonged antibiotic therapy and surgical
drainage.

Educational Objective:
In a smoker with arm pain, cough and weight loss, a mass in the lung
apex is a Pancoast tumor until otherwise proven.

__________________________________________________________ 29
USMLE WORLD STEP 2 CK (INTERNAL MEDICINE) *Block
1*
Q NO 25: A 60-year-old male farmer presents to the office for the A.
evaluation of a slightly painful ulcer on the top of his lower lip. The
ulcer has not healed since he first noticed it three months ago. He
has always been healthy and denies any sexual activity during the
past year. He is afebrile. Physical examination shows a 3x7 mm,
partially encrusted ulcer in the vermilion zone near the moist line
(mucocutaneous junction) of the lower lip, surrounded by a 6x
12mm area of induration. There are no palpable submental or
submandibular lymph nodes. The remainder of the physical
examination is unremarkable. Complete blood count and
differential are normal. Biopsy of the ulcer will most likely to show:
Invasive clusters of spindle cells surrounded by palisaded basal cells
B. Granulomatous inflammation
C. Invasive cords of squamous cells with keratin pearls
D. Shallow fibrin-coated ulceration with an underlying mononuclear
infiltrate
E. Giant cells in a Tzanck preparation

Explanation:
Given the nature of this patients ulcer(i.e., solitary, has not healed
over three months, and located in the keratinized epithelium of the
vermilion zone of the lower lip)the initial differential diagnosis is
broad. The underlying cause may be a malignant, chronic infectious, or
chronic autoimmune process. Nevertheless, the most likely diagnosis of
this patient is squamous cell carcinoma, which is characterized by
invasive cords of squamous cells with keratin pearls. The location of
the ulcer and history of occupational sun exposure make squamous cell
carcinoma the leading diagnosis, as does the epidemiological fact that
the majority of lip cancers are well-differentiated squamous cell c arc
i no ma s.
(Choice A) Basal cell carcinomas are histologically characterized by
invasive clusters of spindle cells surrounded by palisaded basal cells.
Basal cell cancer is rarely found on the lips. In sporadic instances,
it has been reported on/around the upper lip, but hardly ever on the
lower lip.
(Choice B) Granulomas could be found if the ulcer was a gumma of
tertiary syphilis or a local tuberculosis infection; however, these are
unlikely possibilities. A gumma of benign tertiary syphilis is a remote
possibility, but this is very rare in the antibiotic era. Mycobacterium
tuberculosis can produce oral ulcers inoculated by sputum from the
lungs; however, a patient with sufficiently active pulmonary
tuberculosis to produce infective sputum would likely have additional
symptoms such as fever, night sweats, cough, and/or hemoptysis. A
fungal cause is also improbable since the patient is neither
debilitated nor obviously immunocompromised. Venereal disease
ulcerations would have arisen within a few months of the patients last
sexual encounter. Cyclic neutropenia is ruled out by a normal CBC.
(Choice D) Aphthous ulcers are described as shallow, fibrin-coated
ulcerations with underlying mononuclear infiltrates. Aphthae (canker
sores) are recurrent, self-limiting ulcerations of indeterminate
(possibly autoimmune) etiology. These arise in the mucosa of the oral
cavity, but not in surfaces covered by keratinized stratified squamous

30 __________________________________________________________
USMLE WORLD STEP 2 CK (INTERNAL MEDICINE) *Block
1*
epithelium. These are not found in the vermilion zone of the lips or on
the gingiva.
(Choice E) Conceivable viral causes are typically characterized by
multiple and/or simultaneous lesions or signs in other areas of the
body. An exception is a recurrent herpetic cold sore (i.e., labial or
peri-oral)which could arise in the same location as this patients
ulcer, but would be expected to heal within two weeks. A Tzanck
preparation is used to demonstrate the characteristic giant cells (cell
fusions) found in the blister fluid of herpetic infections.

Educational Objective:
The most likely diagnosis of an asymptomatic (i.e., lesion-free),
immunocompetent adult patient with a non-healing, isolated ulcer in the
vermilion zone of the lower lip and a significant history of sun
exposure is squamous cell c arc i no ma.

__________________________________________________________ 31
USMLE WORLD STEP 2 CK (INTERNAL MEDICINE) *Block
1*
Q NO 26: A 58-year-old nurse with node-positive metastatic breast
cancer comes to the office for her monthly follow-up visit. The tumor
is estrogen receptor (ER) and progesterone receptor (PR) positive, and
her whole body bone scan is positive for metastatic disease. She is
being treated with systemic chemotherapy and hormonal therapy
(Tamoxifen). She feels weak with vague muscle joint, and bone pains.
Physical examination reveals a hard, well-defined dominant mass in the
left breast. Mucus membranes are moist. Laboratory studies show the
following results.
Sodium 145 mEq/dL
Potassium 3.9 mEq/dL
Chloride 103 mEq/dL
Bicarbonate 24 mEq/dL
Calcium 11 .3 mg/dL
BUN 18mg/dL
Creatinine 0.8 mg/dL
Glucose 146 mg/dL
Which of the following is the best next step in the management of her
hypercalcemia?

A. Corticosteroid therapy
B. Zoledronic acid therapy
C. Intravenous normal saline
D. Furosemide therapy
E. Intravenous mithramycin

Explanation:
The patients complaints are due to metastatic disease and resultant
hypercalcemia. In patients with malignancy, hypercalcemia is due to the
increased bone resorption and the release of calcium from the bone. The
pathology involves osteolytic metastases with local release of
cytokines and tumor secretion of parathyroid hormone-related protein
(PTHrP).
Bisphosphonates are the drugs of choice for mild to moderate
hypercalcemia. These drugs are relatively nontoxic and more potent than
intravenous saline. In addition, there is increasing evidence that
bisphosphonates may prevent skeletal complications (reduce bone pain,
fracture risk), and perhaps improve survival in patients with multiple
myeloma or breast cancer. According to the clinical practice guidelines
published by the American Society of Clinical Oncology (ASCO),
intravenous Zoledronic acid (ZometaA) is recommended in all women who
have metastatic breast cancer and radiographic lytic bone disease, and
are receiving either hormone therapy or chemotherapy.
(Choice A) Corticosteroids are used in patients with hypercalcemia due
to extra renal production of 1.25 dihydroxy vitamin D (calcitriol) such
as sarcoidosis.
(Choices C and D) IV fluids and furosemide are used in the treatment of
hypercalcemic crisis.
(Choice E) Although IV mithramycin is highly effective, it is rarely
used because of its toxicity.

Educational Objective:
Bisphosphonates are the drugs of choice for mild to moderate
hypercalcemia due to malignancy.

32 __________________________________________________________
USMLE WORLD STEP 2 CK (INTERNAL MEDICINE) *Block
1*
Q NO 27: A 79-year-old woman is brought to the emergency department
due to a suspected cerebrovascular accident on her right side. She
complains of diffuse paresthesias and tingling in her right hand. She did
not lose consciousness. The physical examination reveals normal speech,
symmetric deep tendon reflexes (2/4 on both upper extremities, 2/4 both
patellar reflexes, and 0/4 both Achilles reflexes), a cold right hand, and
undetectable arterial radial pulse. Tinel and Phalens signs are negative.
The rest of the examination is normal. What is the most appropriate next
step in the management of this patient?

A. CT scan of the brain without contrast


B. MRI scan of the brain with diffusion images
C. Doppler of the carotid arteries
D. Schedule for EMGI and nerve conduction studies
E. Immediate vascular surgery consultation for intervention

Explanation:
This patients presentation is typical for an embolus in her right
upper extremity arterial bed, specifically the radial artery. Patients
characteristically have a normal neurological exam with persistent
signs of ongoing ischemia (e. g., paresthesias, change in temperature,
and non-detectable radial arterial pulsations). Immediate
anticoagulation with heparin and surgical intervention (i.e.,
embolectomy) are crucial, because this condition can lead to tissue
death and amputation if not effectively treated within hours.
(Choices A and B) The patients normal neurology examination makes the
diagnosis of stroke unlikely.

Educational Objective:
Immediate anti coagulation with heparin and surgical intervention
(i.e., embolectomy) are crucial to prevent tissue death in a patient
with ongoing ischemia of the limb.

__________________________________________________________ 33
USMLE WORLD STEP 2 CK (INTERNAL MEDICINE) *Block
1*
Q NO 28: A 60-year-old white male presents to the ER with sudden onset of
chest pain, nausea, vomiting, and diaphoresis. He describes this chest pain
as a squeezing sensation, it started 2 hours ago at rest, radiates to jaw, and
is partially relieved by sublingual nitroglycerin. He has a history of 30
pack/year of cigarette smoking. He is a known hypertensive for the past 10
years and suffered a hemorrhagic stroke 6 months ago. His medications
include aspirin, atenolol, and hydrochlorothiazide. On examination, PR:
76/min; BP: 139/79 mmHg; RR: 13/min; Temperature: 37.30(99F). On
auscultation, lungs are clear and heart sounds are normal. Chest x-ray is
normal. EKG shows ST segment elevation of 2mm in leads V3 to V6. These
ST changes persist after sublingual nitroglycerin. Which of the following
steps is the most appropriate regarding the management of this patient?

A. IV tissue plasminogen activator, IV heparin, and aspirin


B. IV tissue plasminogen activator and aspirin
C. IV heparin, IV nitroglycerin, and aspirin
D. IV amiodarone and IV magnesium
E. Immediate coronary angiography and PTCA

Explanation:
Explanation: The above patient is likely to benefit from thrombolytic
therapy, but he has a history of a relatively recent hemorrhagic stroke
(less than 1 year ago). Thrombolytic therapy would put him at a
tremendous risk for intracranial bleeding and is therefore
contraindicated. In such circumstances, immediate coronary angiography
and PICA or stent is the procedure of choice.
Thrombolytic therapy is indicated when the chest pain is suggestive of
Ml and there is ST segment elevation greater than 1 mm in two
contiguous leads after sublingual nitroglycerin administration to rule
out coronary vasospasm. Another indication for thrombolytic therapy is
a new or presumably new left bundle branch block. Thrombolytic therapy
is not indicated for an MI with ST segment depression and it is also
not indicated for unstable angina. In these settings, no proven benefit
has been shown for thrombolytic therapy. Thrombolytic therapy can be
given when ST depression shows a posterior MI, especially in the
setting of an inferior wall MI.
Absolute contraindications to thrombolytic therapy include: active
internal bleeding: history of hemorrhagic stroke anytime, or an
ischemic stroke within past year: known intracranial neoplasm: current
systolic BP greater than 180 or diastolic BP greater than 110: or
suspected aortic dissection.
Thrombolytic therapy with tissue plasminogen activator requires co-
administration of heparin and aspirin for greater benefit. Tissue
plasminogen activator is slightly more effective than streptokinase but
it has a slightly more risk of intracranial bleeding. For
streptokinase, co-administration of heparin is not required.
Treatment with IV heparin, aspirin, and IV nitroglycerin is indicated
in cases of unstable angina and non-Q wave infarcts. In those cases,
thrombolytic therapy has not been shown to be associated with a proven
benefit.
IV amiodarone and IV magnesium are not useful in the setting of Ml
unless arrhythmias or low serum magnesium levels complicate it.
Educational Objective:
34 __________________________________________________________
USMLE WORLD STEP 2 CK (INTERNAL MEDICINE) *Block
1*
Manage a patient with SI segment elevation MI with immediate
angiography and PICA when thrombolytic are contraindicated. Even if the
patient has no contraindications for thrombolytic therapy and a
catheterization laboratory is available in the hospital, or within 30
min of the hospital, PICA with stent placement has been shown to have
better outcomes than thrombolytic therapy in acute ST elevation MI.

__________________________________________________________ 35
USMLE WORLD STEP 2 CK (INTERNAL MEDICINE) *Block
1*
Q NO 29: A 50-year-old woman comes to the emergency department with a
very painful left leg. The problem started one day ago when she
developed pain in the leg. There is no history of trauma to the calf.
She has hypertension and diabetes; both were diagnosed 3 years ago.
There is no past history of arthritis. She is currently taking
captopril, glipizide, and Vitamin E preparation. On physical
examination, her temperature is 38.7 C (103 F); Pulse is 106/min, B.P
130/80, and R.R 20/min. Her left calf is swollen, erythematous, and
extremely tender to touch. There is no overlying crepitus and bullae.
The area of involvement of the calf is about 6X3 cm. There are linear
streaks of erythema along the thigh. The toe webs are fissured and
macerated. The patient tells that itching and stinging in interdigital
webs has been quite disturbing for her. Labs revealed:
Hemoglobin 12 gm/dL
ESR 50 mm/hr
WBC 12,000/cmm
Platelet 250,000/cmm
What is the most probable diagnosis?

A. Cellulitis
B. Deep venous thrombosis
C. Sclerosing panniculitis
D. Ruptured bakers cyst
E. Erysipelas
F. Erysipeloid
G. Necrotizing fascitis

Explanation:
This patient has cellulitis, which is a diffuse infection of deep layers of
skin. Presence of fever, inflammatory signs, no crepitus or bullae and
signs of overlying skin necrosis all suggest that the patient has
cellulitis. This patient also had toe web tinea pedis, which is one of the
most common portals of entry for the microorganisms, causing cellulitis.
Cellulitis of calf is difficult to differentiate from deep venous
thrombosis. However presence of high-grade fever, lymphangitis, absence of
any risk factor such as orthopedic surgery, or prolonged immobilization
make deep venous thrombosis unlikely.
Bakers cyst is a complication of rheumatoid arthritis. This patient has no
history of arthritis.
Necrotizing fasciitis is a deep-seated cellulitis. It should be suspected
in a patient who has evidence of overlying skin necrosis, bullae, with
anesthesia due to destruction of nerves, crepitus due to gas producing
organism, and fever.
Sclerosing panniculitis is acute tender lesion over the medial malleolus.
It usually occurs in a patient with venous stasis of lower limb.
Erysipelas is a superficial cellulitis. It usually affects cheek. The area
involved become erythematous, tender, sharply demarcated and with vesicles
or bullae. There is no element of lymphangitis. Fever is usually present.
Erysipeloid is an edematous, purplish plaque with central clearing. It is
caused by Erysipelothrix insidiosa. It usually occurs on the hands of
fishermen and meat handlers. It is not very painful like cellulitis. Fever
is not present.

Educational Objective:
Painful leg in a lady is frequently tested in USMLE exams. Know how to deal
with various possible scenarios.

36 __________________________________________________________
USMLE WORLD STEP 2 CK (INTERNAL MEDICINE) *Block
1*
Q NO 30: A 65-year-old male with oxygen-dependent chronic obstructive
pulmonary disease, chronic atrial fibrillation, and depression comes into
the Emergency Room, with symptoms of increased dyspnea and
worsening cough pattern. His recent history had been significant for a
gradual worsening of his baseline lung disease over the past month,
which had been treated by his outpatient doctor with increased frequency
of inhaled beta-agonist and azithromycin. This morning he had a severe
shortness of breath that was unresponsive to stacked home nebulizer
treatments. The ER physician notes that the patient is in moderate severe
respiratory distress. His temperature is 37.2 C (99 F), blood pressure is
150/90 mmHg, pulse is 110/min, and respirations are 28/min. Accessory
muscle use was noted. Lung exam shows diffuse rhonchi and wheezing. A
pulse oximetry revealed an oxygen saturation of 80% on room air. His
chest x-ray showed no new infiltrates. His WBC count is 7000/cmm with
normal differential. The ER physician had given nebulization, and the
patient is on 5-liters of oxygen. Which of the following should also be
considered in this patient?

A. Gatifloxacin
B. Methylprednisolone
C. N-acetylcysteine
D. Clarithromycin
E. Aminophylline

Explanation:
This patient has a history of chronic obstructive pulmonary disease
(COPD). His current symptoms are manifestation of an acute exacerbation
of his underlying disease. The acute episodes are typically managed
with ipratropium and albuterol nebulization and systemic steroids. The
most commonly used parenteral steroid is methylprednisolone (Choice B).
N-acetylcysteine is a mucolytic agent, which is no longer used in
exacerbation of COPD, as they are even implicated in worsening
bronchospasm (Choice C).
There is some role of antibiotics, like gatifloxacin and
clarithromycin, in the management of an acute episode of dyspnea in a
patient with severe oxygen dependent COPD: however, this patient has
already been treated with antibiotics, has no fever WBC count is
normal, and there is no infiltrate on chest x-ray. So, due to the
previous facts, having pneumonia or a bacterial infection is unlikely
(Choices A and D).
Aminophylline is proven to be inferior to a combination of
bronchodilator and corticosteroids for the treatment of acute
exacerbation of COPD (Choice E).

Educational Objective:
Acute exacerbation of COPD is treated with a combination of
inhaled/nebulized bronchodilators and systemic steroids.

__________________________________________________________ 37
USMLE WORLD STEP 2 CK (INTERNAL MEDICINE) *Block
1*
Q NO 31: A 55-year-old Asian man with mitral stenosis secondary to
rheumatic heart disease undergoes dental surgery for caries.
Postoperatively, he does well and is discharged home. Two weeks
later, he presents with fever, chills, fatigue, and feels sick. Four out
of four blood culture bottles are positive for gram-positive cocci. An
echocardiogram is performed and shows mitral valve vegetations.
Which of the following is the most likely causative organism of this
patients condition?
A. Group B streptococci
B. Streptococcus mutans
C. Streptococci bovis
D. Staphylococcus epidermis
E. Enterococci

Explanation:
This patient has subacute infective endocarditis (IE) of the mitral
valve secondary to dental surgery. His pre-existing mitral stenosis is
a predisposing factor for infective endocarditis, and bacteremia caused
by dental surgery further increases such risk. Viridans group
streptococci are the most likely cause of endocarditis in native valves
following dental procedures. Four members of the viridans group cause
IE: Streptococcus mitis, S. sanguis, S. mutans, and S. salivarius. S.
mutans also causes dental caries.
(Choice A) Risk factors for Group B streptococcal endocarditis include
diabetes mellitus, carcinoma, alcoholism, hepatic failure, elective
abortion, and intravenous drug use.
(Choice C) S. bovis is a normal inhabitant of the GI tract, and S.
bovis bacteremia is associated with colon cancer. Colonoscopy should be
performed when this organism is isolated from blood cultures.
(Choice D) S. epidermidis is an important cause of prosthetic valve
endocarditis. It is also seen in infants with IE secondary to umbilical
venous catheter infection in neonatal intensive care units.
(Choice E) Enterococci are normal inhabitants of the gastrointestinal
tract and also occasionally colonize the anterior urethra. Enterococcal
endocarditis generally affects older men after genitourinary
manipulation or younger women after obstetric procedures.

Educational Objective:
Viridans group streptococci (most commonly S. mutans) are the most
common cause of endocarditis following dental procedures.

38 __________________________________________________________
USMLE WORLD STEP 2 CK (INTERNAL MEDICINE) *Block
1*
Q NO 32: A 60-year-old white man comes into the Emergency Room with
intensive retrosternal pain that began ten minutes ago. He has never had
such pain before. His past medical history is significant for diabetes
mellitus type 2, controlled with diet. His blood pressure is 150/95 mmHg
and HR is 80/min. You give him one chewable tablet of aspirin and two
sublingual tablets of nitroglycerin with a 5-minute interval. After the
second tablet of nitroglycerin, the pain is greatly relieved. What is the
most important mechanism responsible for pain relief in this patient?

A. Increase in coronary blood flow


B. Increased cardiac contractility
C. Dilation of resistance vessels
D. Dilation of capacitance vessels
E. Change in the activity of baroreceptors

Explanation:
The primary mechanism responsible for the effect of nitroglycerin in
patients with anginal pain is dilation of veins (capacitance vessels).
Increased venous capacitance and venous pooling of the blood lead to
significant decrease in ventricular preload and decrease in heart size.
As the result of these changes, oxygen requirement of the heart greatly
reduces. Nitroglycerin also causes arterial dilation (Choice C) and,
therefore, decreases ventricular after load, but this effect has less
significance in relieving anginal pain. It is unclear whether
nitroglycerin significantly increases coronary blood flow in patients
with obstructive coronary heart disease (Choice A), although it does so
in healthy subjects. Secondary effects provoked by nitroglycerin like
increased heart contractility (Choice B) and reflex tachycardia are due
to change in the activity of baroreceptors (Choice E) in response to
the decrease in blood pressure. These effects increase myocardial
oxygen demand.

Educational Objective:
The main mechanism responsible for pain relief in patients with anginal
pain treated with nitroglycerin is dilation of veins and decrease in
ventricular preload.

__________________________________________________________ 39
USMLE WORLD STEP 2 CK (INTERNAL MEDICINE) *Block
1*
Q NO 33: A 72-year-old male presents with a two-day history of intense
pain in his right ear, along with ear discharge. The pain is so severe that
he is unable to sleep. It radiates to his temporomandibular joint and is
aggravated by chewing. His disease has worsened despite the use of
topical antibiotics. He takes metformin and enalapril. On physical
examination, granulation tissue is noted in the lower part of his external
auditory canal. Cranial nerves are intact. Oropharynx is clear without
exudate. Which of the following is the most likely causative organism of
this patients ear condition?

A. Pseudomonas aeruginosa
B. Staphylococcus aureus
C. Bacteroides species
D. Peptostreptococcus species
E. Aspergillus fumigatus
F. Proteus mirabilis
G. Klebsiella oxytoca
H. Streptococus pneumoniae
I. Hemophilus influenzae
J. Actinomyces israelii

Explanation:
The typical symptoms of malignant otitis externa are ear discharge and
severe ear pain. The pain often radiates to the temporomandibular
joint, and consequently causes pain that is exacerbated by chewing.
Worsening of the disease despite the use of topical antibiotics is an
important indicator of the conditions malignant nature. Examination
shows the presence of granulation tissue in the external auditory
meatus. Diabetes mellitus and other immunosuppressive conditions are
important risk factors. The most frequent causative organism is
Pseudomonas aeruginosa, which is implicated in more than 95% of cases.
(Choices B and E) Aspergillus fumigatus and Staphylococcus aureus are
very rare causes of malignant otitis externa.
(Choices C and D) Anaerobes (e.g., Bacteroides, Peptostreptococcus) are
infrequent causes of simple otitis externa.

Educational Objective:
Suspect malignant otitis externa in any diabetic patient with severe
ear pain, otorrhea, and evidence of granulation tissue in the ear
canal. Pseudomonas aeruginosa is the most frequent cause of malignant
otitis externa.

40 __________________________________________________________
USMLE WORLD STEP 2 CK (INTERNAL MEDICINE) *Block
1*
Q NO 34: A 65-year-old female is complaining of seeing a sudden burst of
flashing lights and blurred vision in her left eye. These symptoms started
this morning. She now sees small spots in her field of vision. She felt like
a curtain came down over her eye. She had a successful cataract
extraction in her left eye 4 months ago. Her vital signs are stable.
Examination shows a sluggish left pupil. Ophthalmoscopy reveals retinal
tears and a grayish-appearing retina. What is the most probable diagnosis?

A. Choroidal rupture
B. Retinal detachment
C. Central retinal artery occlusion
D. Proliferative diabetic retinopathy
E. Exudative macular degeneration

Explanation:
Retinal detachment refers to the separation of the layers of the
retina. It usually occurs in people aged 40-70 years. Patients complain
of photopsia (flashes of light) and floaters (spots in the visual
field.). The most classic description is that of a curtain coming down
over my eyes. Usually, the inciting event occurs months before retinal
detachment. Myopia or trauma can cause retinal breaks, through which
fluid seeps in and separates the retinal layers. In this patient,
ocular trauma most likely occurred due to her cataract surgery.
Ophthalmoscopic examination reveals a grey, elevated retina. Laser
therapy and cryotherapy are done to create permanent adhesions between
the neurosensory retina, retinal pigment epithelium, and choroid.
(Choice A) Choroidal rupture occurs due to blunt ocular trauma.
Examination reveals central scotoma, retinal edema. hemorrhagic
detachment of the macula, subretinal hemorrhage, and crescent-shaped
streak concentric to the optic nerve. The usual complaint is blurred
vision following blunt trauma.
(Choice C) Central retinal artery occlusion (CRAO) is also
characterized by a sudden painless loss of vision in one eye, but its
funduscopic findings differ. Ophthalmoscopy of patients with CRAO
reveals pallor of the optic disc, cherry red fovea, and boxcar
segmentation of blood in the retinal veins.
(Choice D) Proliferative diabetic retinopathy in the initial stage is
asymptomatic. Patients may later complain of decreased visual acuity.
Neovascularization is the hallmark of proliferative diabetic
retinopathy. The other findings are vitreous hemorrhage and macular
edema. These changes may lead to retinal detachment.
(Choice E) Exudative macular degeneration typically presents as
painless, progressive blurring of central vision, which can be acute or
insidious. It occurs bilaterally. Testing reveals central scotoma.
Ophthalmoscopy reveals growth of abnormal vessels in the retinal space.
Sudden visual loss may occur if it is complicated by retinal
detachment.

Educational Objective:
Retinal detachment usually presents with a sudden onset of photopsia
and floaters. The most classic description is that of a curtain coming
down over my eyes.

__________________________________________________________ 41
USMLE WORLD STEP 2 CK (INTERNAL MEDICINE) *Block
1*
Q NO 35: A 27-year-old woman presents to the office due to a significant
amount of hair on her cheeks, chin and upper lips. This symptom
developed over the past two months. Her last menstrual period was 12
weeks ago, but she did not seek medical help because her home
pregnancy test was negative. Her medical history is unremarkable. She
denies taking any medications other than oral contraceptive pills. Physical
examination reveals acne on the forehead and cheeks; there is terminal
hair on the upper lip, cheeks, chin, upper chest and lower abdomen.
Examination of the genitals shows clitoromegaly. The abdominal exam is
normal; the uterus is normal in size. Ultrasound reveals a normal uterus
and ovaries, but there is a left adrenal mass. Which of the following is
most likely to be increased in this patients serum?

A. Luteinizing hormone (LH)


B. Adrenocorticotropic hormone (ACTH)
C. Testosterone
D. Androstenedione
E. Dihydrotestosterone
F. Dehydroepiandrosterone-sulfate

Explanation:
Women produce androgens, such as androstenedione (AS),
dehydroepiandrosterone (DHEA), testosterone (T) and
dehydroepiandrosterone sulfate (DHEA-S). AS DHEA and Tare produced by
the ovaries and adrenals, whereas DHEA-S is predominantly produced in
the adrenal glands only. AS, DHEA and DHEA-S are not true androgens
because they do not interact with the androgen receptor. They can be
converted to testosterone, and overproduction of these hormones can
lead to clinical features of androgen excess (as in this patient).
(Choice B) An increase in ACTH levels in patients with hirsutism are
seen in patients with ectopic or pituitary- dependent Cushings
disease. ACTH increases the production of cortisol, as well as
androgens from the adrenal glands: however, the adrenal glands show
diffuse hyperplasia rather than a discrete adenoma.
(Choices C and D) Testosterone and androstenedione levels may be
elevated in both adrenal and ovarian tumors.
(Choice E) Dihydrotestosterone is the product of the testosterone
conversion in peripheral tissues by 5-alpha- reductase. It is a potent
androgen and interacts with the testosterone receptors. Increased
dihydrotestosterone levels are not specific for androgen-producing
adrenal tumors.

Educational Objective:
Elevated serum DHEA-S levels are specifically seen in patients with
androgen-producing adrenal tumors.

42 __________________________________________________________
USMLE WORLD STEP 2 CK (INTERNAL MEDICINE) *Block
1*
Q NO 36: A 32-year-old man presents for a routine health examination
because itis a requirement for his new job. He has no complaints at this
time. He states that he has no history of medical problems and takes
no medications. He admits to frequent intravenous drug abuse and
intranasal cocaine abuse. Physical examination is unremarkable, with
no evidence of hepatomegaly or icterus. Serologic testing for hepatitis C
is performed. Which of the following is the single most sensitive
serological marker used to screen for HCV infection?

A. Serum aminotransferase
B. HCVRNA
C. Anti-HCV RNA
D. Serum alkaline phosphatase
E. Liver biopsy

Explanation:
Although there has been a recent marked increase in cases of blood
transfusion-related hepatitis C virus (HCV) infection, the risk of
acquiring the virus through transfusion was recently estimated to be
<1:540.000. Intravenous drug abuse continues to account for 50-75% of all
new cases of HCV infection. Since this patient is an intravenous drug
abuser, he is at high risk for infection with HCV. Intranasal cocaine abuse
is also an independent risk factor for HCV infection, thereby increasing
the likelihood that this patient will test positive. Other risk factors
include body piercing, hemodialysis, and multiple sexual partners.
As the first marker of HCV infection to appear in the serum, HCV RNA may be
detected with PCR within days to weeks after infection, depending on the
size of the inoculum. Because this test is the most sensitive serological
marker for diagnosis of HCV infection, it is considered the gold standard.
HCV RNA can be detected before elevation of aminotransferases and before
the appearance of anti-HCV. Moreover, HCV RNA is detectable indefinitely
(continuously or intermittently) in patients with chronic hepatitis C
infection.
(Choice A) The serum aminotransferases are nonspecific markers of hepatic
injury that become elevated approximately 6-12 weeks after exposure to HCV.
These levels tend to wax and wane. Normalization of serum ALT concentration
in a patient with known HCV infection does not indicate that the infection
has resolved.
(Choice C) Commonly used in the diagnosis of hepatitis C infection, anti-
HCV antibodies can be detected with the ELISA technique as early as eight
weeks after exposure. However, some patients may not be positive for
several months after exposure or may never test positive. In addition, this
marker does not distinguish between acute and chronic infections. Anti-HCV
is considered to have moderate sensitivity early in the disease course but
low specificity in patients with elevated gamma globulin levels. If
infection is clinically suspected but not confirmed by the presence of
anti-HCV, then HCV RNA levels should be obtained.
(Choice D) Serum alkaline phosphatase is a better indicator of biliary
tract pathology than of hepatitis.
(Choice E) Liver biopsy is nota screening tool for viral hepatitis, as the
appropriate serological tests are more sensitive and cost-effective. Liver
biopsy is typically of considerable help in determining the etiology of a
hepatic mass or infiltrative liver disease, however.
Educational Objective:
HCV RNA is the single most sensitive serological marker used in screening
for HCV infection.

__________________________________________________________ 43
USMLE WORLD STEP 2 CK (INTERNAL MEDICINE) *Block
1*
Q NO 37: A 55-year-old male patient comes to his primary care physician
for his routine health care examination. He is completely asymptomatic
and has nothing significant in his past medical history. He has 10 pack-
year history of smoking but he does not drink alcohol. His PR: 80/min;
BP: 150/78mm Hg; Temperature: 37.1 C (98.8 F). Labs show
hematocritof43%, blood glucose of 90 mg/dl, serum creatinine of 0.7
mg/dL, serum Na of 140 mEq/L and serum K of 4 mEq/L. Urinalysis and
EKG are unremarkable. His blood pressure readings on subsequent visits
are 155/80 mmHg, 160/78 mmHg and 150/70 mmHg. Life-style
modifications fail to control his blood pressure. Which of the following is
the best initial pharmacological therapy for the control of his blood
pressure?

A. Thiazide diuretics
B. Hydralazine
C. ACE inhibitors
D. Angiotensin receptor blockers
E. Calcium channel blockers

Explanation:
Explanation: This patient is suffering from isolated systolic HTN. The
initial drug of choice, for these patients, is a low- dose thiazide
diuretic.
Long acting dihydropyridine calcium channel blockers are an acceptable
alternative, but they are more expensive.
ACE inhibitors and angiotensin receptor blockers are not first line
agents in the treatment of isolated systolic hypertension without any
other co-morbidities.
Hydralazine is not a first line agent in the treatment of hypertension.

Educational Objective:
Thiazide diuretics are the initial drugs of choice in people with
isolated systolic hypertension.

44 __________________________________________________________
USMLE WORLD STEP 2 CK (INTERNAL MEDICINE) *Block
1*
Q NO 38: A 61-year-old Caucasian male presents with ptosis, diplopia and
limb weakness. These symptoms worsen in the evening and with exercise,
and improve with rest. He also has fatigue, which is worse in the evening.
He denies any tingling or numbness. On examination, he cannot sustain an
upward gaze, and his eyelids tend to drift downward. Injection of
edrophonium quickly restores power, and allows him to maintain an
upward gaze. Which of the following is the best initial treatment for this
patient?

A. Treatment with pyridostigmine


B. Treatment with edrophonium
C. Treatment with atropine
D. Treatment with prednisolone
E. Treatment with intravenous immunoglobulins
F. Plasmapheresis

Explanation:
There are three treatment options available for the treatment of
myasthenia gravis. These include anticholinesterases, immunosuppressive
agents and thymectomy. Anticholinesterases provide symptomatic benefit,
but do not induce remission. Immunosuppressive agents and thymectomy
may induce remission. The choice of treatment depends on the patients
age and the clinical scenario.
Oral anticholinesterase is usually the initial treatment of choice for
myasthenia gravis. It produces its useful effect by increasing the
availability of acetylcholine at the neuromuscular junction, where the
number of acetylcholine receptors is reduced due to acetylcholine
receptor antibodies. Pyridostigmine or neostigmine is used for
treatment purposes. Side effects include abdominal cramps,
fasciculations and muscular weakness.
(Choice B) Edrophonium is a short acting anti-cholinesterase, and is
used for diagnostic purposes only.
(Choice C) Atropine is an anticholinergic agent that can be used to
prevent muscarinic side effects of anticholinesterase therapy in
patients with myasthenia gravis.
(Choice D) Immunosuppressive agents that are used in the treatment of
myasthenia gravis include prednisolone, azathioprine and cyclosporine.
Corticosteroids are used in (1) patients who are over 60 years of age,
and (2) patients who show a poor response to anti cholinesterases and
have already undergone thymectomy.
(Choice F) Plasmapheresis removes acetylcholine receptor antibodies
from the circulation. It is used in seriously ill patients when other
treatments are not effective or contraindicated. Its effect is
transient and cannot be used on a long- term basis. Other indications
of plasmapheresis include (1) stabilization of the patient before
thymectomy, and (2) myasthenia crisis.
(Choice E) Intravenous immunoglobulins act by an unknown mechanism, and
are used in the same setting as plasmapheresis.

Educational Objective:
Oral anticholinesterase is usually the initial treatment of choice for
myasthenia gravis. All patients should be evaluated for a thymectomy.

__________________________________________________________ 45
USMLE WORLD STEP 2 CK (INTERNAL MEDICINE) *Block
1*
Q NO 39: You are called by the hospice nurse to evaluate a 74-year-old
man who refuses to eat. He has terminal colon cancer, which was
diagnosed 8 months ago and has metastasized to his liver. He has
already gone through chemotherapy with 5-Flourouracil + oxaliplatin.
During the past 4 months, his condition has gotten extremely worse. He
has lost 5Olbs (21.6kg), and currently appears totally wasted. As you
approach him, he says, I dont feel like eating anything, and I dont
want anymore procedures, but Ill drink any pill that can make me feel
better. His abdomen is distended with positive fluid thrills. His liver has
several palpable nodules. What is the best pharmacological agent of
choice for this patients anorexia?

A. Megestrol acetate
B. Dexamethasone
C. Dimenhydrinate
D. Ondansetron
E. Metoclopramide

Explanation:
Anorexia is a common problem of terminally ill patients. In palliative
care, the rule-of-thumb is that it is always better to have oral rather
than parenteral nutrition. All efforts need to be made to encourage the
patient to eat adequate meals (e.g., giving the patient his preferred
meal, presenting the food attractively, giving small and frequent
meals). The next step is pharmacologic. The drug of choice for cancer-
associated anorexia is megestrol acetate, which is a synthetic
progestin with progestational effects similar to those of progesterone.
Itis currently used as an appetite stimulant in patients with advanced
malignancies. Prednisone may be used in addition to this drug.
(Choice B) Dexamethasone is not used for malignancy-associated
anorexia.
(Choices C, D and E) Dimenhydrinate, ondansetron, and metoclopramide
are anti-emetics which are usually given to patients who are undergoing
chemotherapy.

Educational Objective:
Megestrol acetate is the drug of choice for cancer-associated anorexia.

46 __________________________________________________________
USMLE WORLD STEP 2 CK (INTERNAL MEDICINE) *Block
1*
Q NO 40: A 69-year-old Caucasian male undergoes open-heart surgery for
CABG and aortic valve replacement. His surgery is uneventful. He is
extubated and discharged to the step down unit on postoperative day 2. At
night, the nurse suddenly calls you because the patient is feeling dry and
his tele monitor is abnormal. His BP is 70/30mm Hg and the respiratory
rate is 22/min. His rhythm strip is shown below. The next step in his
management is:

A. Intravenous amiodarone
B. Endotracheal intubation
C. Cardioversion
D. Intravenous digoxin
E. Intravenous verapamil

Explanation:
When atrial fibrillation (absent P waves and irregular heart rate) is
associated with hemodynamic compromise, the role of medical therapy is
limited. Direct current cardioversion should be the treatment of
choice. Cardioversion is indicated when the arrhythmia is poorly
tolerated and or when one anticipates that sinus rhythm can be
maintained, as when atrial fibrillation has been of short duration, and
when it occurs in the absence of marked atrial enlargement or severe
mitral stenosis. Atrial fibrillation usually requires higher energy for
conversion (100-200 joules) (Choice C).
(Choice A) Amiodarone is a class 3 anti arrhythmic agent, which is
excellent for control of both atrial and ventricular arrhythmias. The
drug is used when the atrial arrhythmia is stable and there is no
hemodynamic compromise. Because the drug can also cause hypotension,
its use in hypotensive patients is a relative contraindication. Once
the patient has been cardioverted, amiodarone is an excellent drug to
maintain normal sinus rhythm (NSR).
(Choice B) In the above patient, endotracheal intubation is not
indicated. The atrial fibrillation is the cause of lethargy and low
blood pressure and once treated, most patients immediately recover.
(Choice D) Digoxin is a poor drug for treatment of acute atrial
fibrillation with hemodynamic compromise. Digoxin does not act for
hours and the patient has to be loaded with the drug beforehand. The
drug is more useful for chronic atrial fibrillation when the
ventricular rate is already controlled.
(Choice E) Calcium channel blockers are excellent for treating atrial
fibrillation when there is no hemodynamic compromise evident. In the
presence of hemodynamic compromise, these drugs should be withheld, as
they are also negative inotropes.

Educational objective:

__________________________________________________________ 47
USMLE WORLD STEP 2 CK (INTERNAL MEDICINE) *Block
1*
When atrial fibrillation is associated with hemodynamic compromise,
cardioversion is the treatment of choice.

48 __________________________________________________________
USMLE WORLD STEP 2 CK (INTERNAL MEDICINE) *Block
1*
Q NO 41: A 76-year-old male comes to the emergency room with sudden
onset of severe chest pain. The pain is 10/10 in severity, tearing in
quality, and radiates to the back. He is a known diabetic for 10 years and
is hypertensive for 8 years. His medications include insulin, metoprolol
and hydrochlorothiazide. His vitals are, PR: 86/min; BP: 166196 mmHg in
his right arm and 136/70 in his left arm; Temperature: 37 C (98.6F); PR:
14/min. On auscultation, lung fields are clear and heart sounds are
normal without any murmurs. Chest x-ray and EKG are normal. Bases on
these findings, which of the following is the most likely diagnosis of this
patient?

A. Angina pectoris
B. Myocardial infarction
C. Acute pericarditis
D. Aortic dissection
E. Pulmonary embolism

Explanation:
Explanation: This patient is most likely suffering from aortic
dissection. The most frequent predisposing condition for aortic
dissection is hypertension. Sudden onset of severe tearing pain, with
radiation to the back, is a typical presentation of aortic dissection.
The finding of a difference of more than 30 mmHg in the blood pressure
readings between two arms is another important clue for the diagnosis
of aortic dissection.
Pain of IHD usually feels like pressure and it typically radiates to
the jaw, left shoulder or left arm. Pain of angina occurs with exertion
and is relieved with rest and sublingual nitroglycerin. Pain of Ml
occurs at rest and is not fully relieved by sublingual nitroglycerin.
EKG usually provides evidence for ischemic heart disease with T-wave
inversion in angina pectoris, and ST segment elevation or ST segment
depression in cases of Ml. The nature of pain in the above patient with
absent EKG changes makes the diagnosis of angina, or Ml, very unlikely.
Pain in acute pericarditis is pleuritic, is worsened by deep breathing
and coughing and is alleviated by leaning forward. EKG may show ST
segment elevation that is concave upwards without any reciprocal
changes.
Pulmonary embolism usually presents with sudden onset of dyspnea, chest
pain, tachycardia, and tachypnea. There is usually a history of risk
factors like immobilization, surgery of hip or femur etc. that
predisposes the development of pulmonary embolism.

Educational Objective:
Suspect aortic dissection as a cause of tearing chest pain in the
setting of HTN and BP difference in the 2 arms.

__________________________________________________________ 49
USMLE WORLD STEP 2 CK (INTERNAL MEDICINE) *Block
1*
Q NO 42: A 30-year-old woman comes to the physician because of a 2-day
history of pen orbital edema and abdominal distention. She has no other
complaints. Her temperature is 37.1 C (98.9F), blood pressure is
125/75mm Hg, pulse is 80/min, and respirations are 14/min. Examination
shows ascites. Urinalysis shows proteinuria: 24-hour urinary protein
excretion is 4 g/day, total serum protein is 5 g/dL and serum albumin is
2.5g/dL. A diagnosis of nephrotic syndrome is made. Renal biopsy is
performed. She is started on diuretics and her salt and protein intake is
restricted. Her edema begins to improve. However, the patient suddenly
develops severe abdominal pain, fever, and gross hematuria. Which of the
following is the most likely diagnosis that will be revealed by renal biopsy?

A. Minimal change disease


B. Systemic amyloidosis
C. Focal segmental glomerulosclerosis
D. Membranous glomerulonephritis
E. Diabetic nephropathy

Explanation:
Explanation: Renal vein thrombosis is an important complication of
nephrotic syndrome. Antithrombin Ill is lost in the urine and puts
patients at an increased risk of venous and arterial thrombosis. Renal
vein thrombosis presents with sudden onset of abdominal pain, fever and
hematuria. It can occur in any form of nephrotic syndrome, but it is
most common with membranous glomerulonephritis, which is the most
common cause of nephrotic syndrome in adults (Choice D).
(Choices A, B, C and E) Minimal change disease, systemic amyloidosis,
focal segmental glomerulosclerosis and diabetic nephropathy can all
cause nephrotic syndrome and subsequent renal vein thrombosis; however,
this complication is less frequent with these disorders.

Educational Objective:
Renal vein thrombosis is an important complication of nephrotic
syndrome, which is most commonly caused by membranous
glomerulonephritis in adults.

50 __________________________________________________________
USMLE WORLD STEP 2 CK (INTERNAL MEDICINE) *Block
1*
Q NO 43: A 26-year-old woman presents with a one-week history of
dysuria and increased urinary frequency. She admits to having multiple
sexual partners in the past. Her temperature is 37.1 C (98.9 F), blood
pressure is 110/70 mm Hg, pulse is 68/min, and respirations are 15/min.
Examination shows suprapubic tenderness. Mucopurulent discharge is
observed at the urethral os. Urinalysis shows:
Blood Negative
Glucose Negative
Ketones Negative
Leukocyte esterase Positive
Nitrites Negative
WBC 40-50/hpf
RBC 1-2/hpf
Bacteria None
Urine culture after 24 hours: <100 colonies/mL. Which of the following
is the most likely diagnosis?

A. Acute pyelonephritis
B. Acute bacterial cystitis
C. Chlamydial urethritis
D. Gonococcal urethritis
E. Trichomonal vaginitis

Explanation:
The diagnosis of chlamydial urethritis is usually suggested by the
presence of mucopurulent urethral discharge and history of multiple
sexual partners. Dysuria and urinary frequency can occur. Urinalysis
reveals absent bacteriuria. Urine culture shows less than 100
colonies/mL.
(Choice A) Patients with acute pyelonephritis appear toxic and present
with fever, nausea, vomiting, flank pain, dysuria, and costovertebral
tenderness. Urinalysis shows bacteriuria and pyuria. Urine culture
reveals greater than 10,000 colonies/mL.
(Choice B) Patients with acute bacterial cystitis present with dysuria,
urinary frequency, suprapubic discomfort and urinary findings of
bacteriuria and pyuria. Mucopurulent cervical discharge is not found.
Urine culture shows colonies in excess of 1000/mL.
(Choice D) Gonococcal urethritis is less common than chlamydial
urethritis. The urethral discharge is purulent (rather than
mucopurulent), and Gram stain usually reveals the causative organisms.
(Choice E) The usual features of vaginitis are vaginal discharge,
pruritus and dyspareunia. Increased urinary frequency is unusual.

Educational Objective:
Chlamydial urethritis is suggested by mucopurulent urethral discharge,
absent bacteriuria, and history of multiple sexual partners.

__________________________________________________________ 51
USMLE WORLD STEP 2 CK (INTERNAL MEDICINE) *Block
1*
Q NO 44: A 64-year-old male with a history of rheumatic fever presents to
you with general malaise and funny heart rhythm for the past 2 weeks. He
had an ECHO done last year, which revealed that he had mild degree of
mitral stenosis. He was managed medically with diuretics. The blood
pressure is 150/85mm Hg. His current EKG is shown below. The next step
in his management is:

A. Cardioversion at 5O joules
B. Lidocaine
C. Flecainide
D. Carotid massage
E. Diltiazem

Explanation:
Atrial fibrillation is due to the presence of macro re entry circuits.
It is characterized by lack of organized atrial activity, with atrial
fibrillatory waves, i.e. irregular undulation of the base line visible
on the surface EKG. No discrete P wave is seen and the QRS is
irregularly spaced. Treatment of atrial fibrillation in a
hemodynamically stable patient is different from one who is unstable.
Anytime if the patient is hemodynamically unstable (low BP, patient not
responding to commands) treatment of choice is electrical
cardioversion. If the patient is hemodynamically stable, then you have
to think whether it is an acute process? or a chronic process? If it is
an acute process then you can choose to convert the patient to sinus
rhythm by either cardioversion or you can keep the patient under rate
control. If it is a chronic process then it is best managed with rate
control along with anticoagulation.
Cardioversion can be either electrical or chemical. Electrical
cardioversion (Choice A) is indicated in hemodynamically unstable
patients. Electrical cardioversion should not be performed without 3-4
weeks of anticoagulation in chronic atrial fibrillation (>48 hours), as

52 __________________________________________________________
USMLE WORLD STEP 2 CK (INTERNAL MEDICINE) *Block
1*
the risk of embolization is high. The required joules are about 1 00-
200 joules. Atrial fibrillation is generally more resistant to convert
to normal sinus rhythm than atrial flutter and the energy required is
much higher. Patients do require intravenous sedation before
cardioversion. If you choose to do chemical cardioversion the best
drugs would be class Ill agents (amiodarone, sotalol, ibutilide etc.).
In the above hemodynamically stable patient, the best treatment is to
administer intravenous diltiazem. He has these symptoms for about 2
weeks. So, he most likely has chronic atrial fibrillation. Since he is
not anticoagulated cardioversion should not be attempted. Within a few
minutes to hours, the rate can be controlled with either IV diltiazem
(Choice E) or beta-blockers (IV metoprolol). Digoxin is a good agent
for patients with heart failure (systolic dysfunction) and atrial
fibrillation.
(Choice B): Lidocaine is a type, anti arrhythmic agent used mainly in
the treatment of ventricular arrhythmias. It has no role in the
treatment of atrial fibrillation. Lidocaine is given intravenously or
may even be administered via the endotracheal tube (when no IV access
exists).
(Choice C): Flecainide has been associated with sudden deaths and is
rarely used in the clinical practice. It is a last resort for resistant
arrhythmias.
(Choice D): The role of carotid massage has declined in the last
decade. It has no role in the management of atrial fibrillation.
Carotid massage has led to release of emboli from the carotid plaques
leading to strokes and the maneuver is now not recommended.

Educational Objective:
Anytime if the patient is hemodynamically unstable, treatment of choice
is electrical cardioversion. If the patient is hemodynamically stable
then you have to think whether it is an acute process? Or a chronic
process?. If itis an acute process then you can choose to convert the
patient to sinus rhythm by either cardioversion or you can keep the
patient under rate control. If it is a chronic process then it is best
managed with rate control along with anti coagulation. Rate control is
best achieved with diltiazem or metoprolol.

__________________________________________________________ 53
USMLE WORLD STEP 2 CK (INTERNAL MEDICINE) *Block
1*
Q NO 45: A 25-year-old man presents to the emergency department with a
two-day history of high-grade fever with chills. He admits to using IV drugs
for the last two years. Physical examination reveals a holosystolic murmur
at the cardiac apex. Ophthalmoscopy reveals exudative, edematous,
hemorrhagic lesions on the retina. These lesions are not documented in his
previous health visit for sore throat one month ago. Blood cultures are
positive for Staphylococcus aureus. Transesophageal echocardiography
shows vegetations on both the tricuspid and mitral valves. Which of the
following explains the pathophysiology of the retinal lesions in this patient?

A. Coagulopathy
B. Immune vasculitis
C. Septic emboli
D. Vasospasm

Explanation:
The pathophysiologic consequences and clinical manifestations of
infective endocarditis (IE) can be explained by the following:
1. Cytokine production, which is responsible for constitutional
symptoms (e.g., fever).
2. Embolization of vegetation fragments, which lead to infection or
infarction of remote tissues (e.g., pulmonary and splenic infarcts).
3. Hematogenous spread of infection during bacteremia.
4. Tissue injury from immune complex deposition or immune response to
deposited bacterial antigens.
Roth spots are exudative, edematous lesions on the retina. They appear
as oval hemorrhages with pale centers. The underlying pathophysiology
is an immune vasculitis. These lesions are an infrequent finding in
patients with IE. They have also been noted in patients with collagen
vascular disease and hematologic disorders, such as severe anemia.
Immune activation is also thought to be responsible for Oslers nodes,
I-associated glomerulonephritis, and the rheumatologic manifestations
of IE. Oslers nodes are violaceous nodules on the pulp of the fingers
and toes thought to be caused by immune complex deposition.
(Choices A and D) Coagulopathy and vasospasm are not responsible for
the pathophysiology of Roth spots.
(Choice C) Janeway lesions are macular, erythematous, blanching, non-
painful lesions on the palms and soles. They are due to septic emboli.
Subcutaneous abscesses are seen on histologic examination.

Educational Objective:
Immunocomplex disease is primarily responsible for IE-associated
glomerulonephritis, Roth spots, and Oslers nodes. Janeway lesions
result from septic embolism.

54 __________________________________________________________
USMLE WORLD STEP 2 CK (INTERNAL MEDICINE) *Block
1*
Q NO 46: A 22-year-old man comes to the urgent care clinic with a one-
week history of fever, sore throat. and malaise. He has tried several
over-the-counter products with partial relief. His temperature is 38.8C
(102 F), pulse is 110/min, respirations are 18/min, and blood pressure
is 130/70 mm Hg. Oropharyngeal examination reveals palatal petechiae
with streaky hemorrhages and blotchy, red macules. The tonsils are
enlarged and covered with whitish exudate. Mild jaundice is present.
Enlarged lymph nodes are palpable posterior to the sternocleidomastoid
muscle bilaterally. Axillary lymphadenopathy is also present. Abdominal
examination reveals normal bowel sounds and mild hepatosplenomegaly.
His complete blood count is shown below:
Hemoglobin 14.0 g/L
MCV 88 fl
Platelets 140,000/mm
Leukocyte count 14,500/mm
Neutrophils 33 %
Lymphocytes 66 %
Eosinophils 1 %
Which of the following is commonly associated with this patients
condition?

A. Bronchopneumonia
B. Autoimmune hemolytic anemia
C. Splenic infarction
D. Hepatocellular carcinoma
E. Dilated cardiomyopathy
F. Glomerulonephritis

Explanation:
The patient is suffering from infectious mononucleosis (IM). IM is
characterized by fever, sore throat, toxic symptoms, and lymphadenopathy.
The characteristic distribution of lymph node involvement in IM is
symmetric and involves the posterior cervical chain of lymph nodes more
frequently than the anterior chain. Inguinal and axillary lymphadenopathy
can also be present.
Other physical findings include pharyngitis, tonsillitis, and tonsillar
exudates. Mild palatal petechiae may be found, but this is non-specific,
and is also found in patients with streptococcal pharyngitis. Tonsillar
enlargement can cause airway compression. Hepatitis and jaundice are
present in a small percentage of cases.
Patients with IM can develop autoimmune hemolytic anemia and
thrombocytopenia. Hemolytic anemia results from anti-I antibodies against
red blood cells and is usually Coombs-test positive.
(Choice A) Bronchopneumonia can occur as a superinfection in patients with
IM. However, it is very unusual.
(Choice C) Splenic infarction does not occur in IM. On the other hand,
splenic rupture is one of the feared complications of IM, and trauma
precedes half of such cases. Contact sports, such as soccer, should be
avoided.
(Choice D) Hepatocellular carcinoma is not associated with EBV infection.
(Choice E) Dilated cardiomyopathy is a possible complication of
Corynebacterium infection, which presents as a sore throat with
pseudomembrane formation. It is not seen with EBV infection.
(Choice F) Glomerulonephritis is a complication of streptococcal
pharyngitis.

__________________________________________________________ 55
USMLE WORLD STEP 2 CK (INTERNAL MEDICINE) *Block
1*
Educational Objective: Always suspect infectious mononucleosis (IM) in a
young patient with a sore throat and nonspecific symptoms. Patients with IM
can develop autoimmune hemolytic anemia and thrombocytopenia.

56 __________________________________________________________

Vous aimerez peut-être aussi